Torts Rule Statements

62
Torts Outline CH. 1 Development of Liability Based Upon Fault A tort is a civil wrong for which the law provides a remedy. This area of law imposes duties on persons to act in a manner that will not injure other persons. A person who breaches a tort duty has committed a tort and be liable to pay damages in a lawsuit brought by a person injured because of that tort. Annonymous: A man whose actions directly cause an injury to another, is liable to the other person even though his actions were not unlawful, and he did not have the intent to cause the injury to another party. Weaver v. Ward: A defendant is liable for all his actions causing injury unless the injury was caused utterly without his fault. (moving away from strict liability) Brown v. Kendall: An individual, who during the commission of a lawful act uses ordinary care, is not liable for injuries caused to another party as a result of his act. (moving farther away from strict liability) Cohen v. Petty: A party is not liable in negligence for the injury caused by his unforeseeable and involuntary actions. Spano v. Perini Corp.: An individual who has sustained property damage caused by blasting may recover for his damages without a showing of negligence by the defendant due to policy reasons. Based on determining: when a party has engaged in a lawful but dangerous activity, who should bear the cost of any losses occurring as a result of the activity, the innocent part or the party who has chosen to commit the lawful, yet dangerous act. Clearly the latter should. Note that in this case the defendant were not negligent, which means that without strict liability as a theory for recovery, the plaintiff would never have recovered for his damages in negligence. CH.2 Intentional Interference with Person or Property 1. Intent Intent is defined two ways (1) with the purpose or knowledge of causing a consequence, or (2) where the actor has the knowledge that a certain consequence is substantially certain to follow. Garratt v. Dailey: A party is liable for battery when he is substantially certain that his act will result in harmful or

Transcript of Torts Rule Statements

Page 1: Torts Rule Statements

Torts Outline

CH. 1 Development of Liability Based Upon Fault

A tort is a civil wrong for which the law provides a remedy. This area of law imposes duties on persons to act in a manner that will not injure other persons. A person who breaches a tort duty has committed a tort and be liable to pay damages in a lawsuit brought by a person injured because of that tort.

Annonymous: A man whose actions directly cause an injury to another, is liable to the other person even though his actions were not unlawful, and he did not have the intent to cause the injury to another party.

Weaver v. Ward: A defendant is liable for all his actions causing injury unless the injury was caused utterly without his fault. (moving away from strict liability)

Brown v. Kendall: An individual, who during the commission of a lawful act uses ordinary care, is not liable for injuries caused to another party as a result of his act. (moving farther away from strict liability)

Cohen v. Petty: A party is not liable in negligence for the injury caused by his unforeseeable and involuntary actions.

Spano v. Perini Corp.: An individual who has sustained property damage caused by blasting may recover for his damages without a showing of negligence by the defendant due to policy reasons. Based on determining: when a party has engaged in a lawful but dangerous activity, who should bear the cost of any losses occurring as a result of the activity, the innocent part or the party who has chosen to commit the lawful, yet dangerous act. Clearly the latter should. Note that in this case the defendant were not negligent, which means that without strict liability as a theory for recovery, the plaintiff would never have recovered for his damages in negligence.

CH.2 Intentional Interference with Person or Property

1. Intent

Intent is defined two ways (1) with the purpose or knowledge of causing a consequence, or (2) where the actor has the knowledge that a certain consequence is substantially certain to follow.

Garratt v. Dailey: A party is liable for battery when he is substantially certain that his act will result in harmful or offensive touching. To maintain an action in battery, it is not enough that the defendant commits the act itself intentionally, even though the act may cause a great risk of harm, the defendant must know or realize with a substantial certainty that his act will bring about the contact or harmful touching. Where a reasonable person would believe that a particular result was substantially certain to follow, he will be held in the eyes of the law as though he had intended it.

Spivey v. Battaglia: A party who acts with knowledge and substantial certainty that a particular result will follow is liable for all results flowing from his acts regardless of how unforeseeable or unreasonable. Where a reasonable person would believe that a particular result was substantially certain to follow, he will be held in the eyes of the law as though he had intended it.

Ranson v. Kitner: Good faith mistake is not a defense to intentional torts where the defendant intended the consequence of his act. (confusing neighbor’s dog for wolf and killing it)

McGuire v. Almy: An insane individual who is capable of forming the intent to strike another or commit a harmful act and acts upon that intent is liable to that individual for the injuries suffered. The fact that insanity may have been the cause of the intent is irrelevant.

Talmage v. Smith: An individual who intends an act or consequence against one party and instead injures a third party is liable to the third party for the injuries suffered. Transferred intent: A doctrine in tort law that provides that if defendant intends to harm A but instead harms B, the intent is said to be transferred to the harm befalling the actual victim as far defendant’s liability to B is concerened. This is the doctrine of transferred intent, it holds that as long as the defendant

Page 2: Torts Rule Statements

held the necessary intent with respect to one person, he will be held to have committed an intentional tort against any other person who happens to be injured.

A person can be charged with an intentional tort even if they intended to commit another one.

2. Battery

Battery is intentional infliction of harmful or offensive bodily contact. At common law, battery is the tort of intentionally and voluntarily bringing about an unconsented harmful or offensive contact with a person or to something closely associated with them (e.g. a hat, a purse). Unlike assault, battery involves an actual contact. The contact can be by one person (the tortfeasor) of another (the victim), or the contact may be by an object brought about by the tortfeasor.

Cole v. Turner: Intentional touching of another in an unreasonable and violent manner is battery. In the common law rule, anger or violence was a required element for battery.

Wallace v. Rosen: In a crowded world, a certain amount of personal contact is inevitable and must be accepted. Absent expression to the contrary, consent is assumed to all those ordinary contacts which are customary and reasonably necessary to the common intercourse of life, such as a tap on the shoulder to attract attention, a friendly grasp of the arm, or a casual jostling to make a passage. The time and place, and the circumstances under which the act is done, will necessarily affect its unpermitted character, and so will the relations between the parties. The test is what would be offensive to an ORDINARY person not unduly sensitive as to personal dignity.

Fisher v. Carrousel Motor Hotel, Inc.: The snatching of an object closely attached to an individual constitutes battery even though there was no actual physical contact to the individual’s body. Unpermitted and intentional contacts with anything so connected with the body as to be customarily regarded as part of the other’s person and therefore as partaking of its inviolability is actionable as an offensive contact with his person.

RESTATEMENT (SECOND) OF TORTS (1965)

§ 13 Battery: Harmful Contact

An actor is subject to liability to another for battery if (a) he acts intending to cause a harmful or offensive contact with the person of the other or a third person, or an imminent apprehension of such a contract, and (b) a harmful contact with the person of the other directly or indirectly results.§18 Battery: Offensive Contact

(1) An actor is subject to liability to another for battery if (a) he acts intending to cause a harmful or offensive contact with the person of the other or third person, or an imminent apprehension of such a contact, and (b) an offensive contact with the person of the other directly or indirectly results. (2) An act which is not done with the intention stated in subsection (1, a) does not make the actor liable to the other for a mere offensive contact with the other’s person although the act involves an unreasonable risk of inflicting it and therefore, would be negligent or reckless if the risk threatened bodily harm.

Elements of Battery

1. An affirmative acta. the act must be voluntaryb. Involuntary/unconscious acts do not qualify

2. Intenta. Purpose/ knowledge to cause harmful or offensive touching;b. Substantial certainty that substantial or offensive touching will occur.

3. Harmful or Offensive Touchinga. Objective standard: is the person harmful/ offensive to a reasonable person?

Page 3: Torts Rule Statements

b. Extreme sensitivities of the individuals will not be considered unless the defendant knew of the sensitivities.

c. Plaintiff need not be aware of the touching at the time that it occurs.d. Touching must be with the individual’s person (body) or any object which may be

considered a natural extension of the individual’s body. 4. Plaintiff must prove that defendant’s voluntary act caused the harmful touching.

3. Assault

In common law, assault is the tort of acting intentionally and voluntarily causing the reasonable apprehension of an immediate harmful or offensive contact. No actual contact must be involved. Words in themselves, no matter how threatening, do not constitute an assault. Words must be followed by some overt act, no matter how slight, that adds to the threatening character of the words.

I de S et ux. v. W de S: A defendant may be liable in assault of a plaintiff, even though there has been no actual physical invasion of the plaintiff by the defendant. (D threw the hatchet at the P)

Western Union Telegraph Co. v. Hill: The actual ability of the defendant to cause harmful or offensive touching is not a requirement for actual assault. Although it was a jury question, the case was reversed on grounds that Sapp had not acted within the scope of his employment.

Elements of Assault

1. A voluntary/affirmative overt acta. Words alone are not enoughb. Physical act alone is enoughc. Words may undo act/conduct

2. Intenta. Purpose/ knowledge to cause harmful apprehension of harmful or offensive touching; orb. Substantial certainty that harmful apprehension of harmful or offensive touching will occur;

orc. Transferred intent applies across torts (especially between battery and assault) and targets.

3. Imminent Apprehensiona. Plaintiff must have been aware and must have perceived the defendant’s conduct. (unlike

battery);b. The apprehension must be imminent: threat of future harm is not sufficient;c. Plaintiff’s apprehension must be reasonable (objective standard).

4. Causation: Plaintiff must prove that the defendant’s voluntary act caused the plaintiff’s apprehension.

4. False Imprisonment

Direct and intentional restraint of the physical liberty of another through mental or physical boundaries without adequate legal justification. The two key elements of false imprisonment are (1) the restraint of an individual against his will and (2) the unlawfulness of such restraint. The individual may be restrained by acts or merely by words which he fears to disregard.

Big Town Nursing Home, Inc. v. Newman: A defendant is liable for false imprisonment when the defendant has prevented the plaintiff from leaving a certain limited area without legal justification.

Parvi v. City of Kingston: A defendant is not liable for falsely imprisoning the plaintiff when the latter was not aware of the confinement at the time it occurs.

Hardy v. Labelle’s Distributing Co.: A person cannot maintain an action for false imprisonment when no threat of force was used to compel that person to stay against his/her will.

Enright v. Groves: A police officer is liable for arrest when he arrests an individual without a warrant or probable cause that an offense has been committed by an individual.

Page 4: Torts Rule Statements

Whittaker v. Sandford: Confinement of an individual to a bounded area, without means of egress, can result in false imprisonment even if there is no actual physical restraint used upon the individual.

Elements of False Imprisonment

Affirmative act or omissiona. Physical confinement: actual confinement or restriction of one’s physical mobility.b. Confinement through taking of a personal property without which he/she can not leave.c. Mental confinement: fear of harm forces the individual to remain within the limited space.d. Failure to provide the fear of egress (way out).

Intent1. Purpose/knowledge to confine; or2. Substantial certainty that the act will cause confinement; or3. Transferred intent applies across torts.

Limited Space1. D act must confine P to a limited space with confined boundaries.2. No false imprisonment when an individual is prevented from entering an area or space

Awareness 1. P must be aware if his or her confinement at the time of the confinement. (Restatement

(Second) of Torts § 42 has taken the position that there is no liability for intentionally confining another unless the person physically restrained knows of the confinement or is harmed by it.)

2. Exception: some courts waive the awareness requirement if the P has suffered physical injury during or because of confinement.

No reasonable means of escape: There is no false imprisonment:1. There is a reasonable means of escape, and2. The P is aware of it at the time of confinement

Individual must be held against his or her free will1. mere obligation to stay is not enough2. submission to stay due to persuasion of another is not sufficient

C. False Arrest: arrest and custody by a person who claims, but does not have legal authority. False arrest may lead to false imprisonment if other elements of the torts are satisfied. No action of the person arrested actually committed the crime for which he/she is arrested.

Difference between false imprisonment and false arrest: A false arrest must be perpetrated by one who asserts that he or she is acting pursuant to legal authority, whereas a false imprisonment is any unlawful confinement.

5. Intentional Infliction of Emotional Distress

The intentional or reckless infliction of severe mental and emotional distress through extreme and outrageous conduct.

§ 46 Outrageous Conduct Causing Severe Emotional Distress (1) One who by extreme and outrageous conduct intentionally or recklessly causes severe emotional

distress to another is subject to liability for such emotional distress, and if bodily harm to the other results from it, for such bodily harm.

State Rubbish Collectors Ass’n v. Siliznoff: One who causes severe and emotional distress to another is liable for such emotional distress and for bodily harm resulting from it.

Slocum v. Food Fair Stores of Florida: Conduct that causes mere emotional distress is not severe enough to allow recovery based on an independent cause of action.

Page 5: Torts Rule Statements

Harris v. Jones: An individual must suffer a severely disabling emotional response to the tortfeasor’s conduct in order to recover damages for intentional infliction of emotional distress.

Taylor v. Vallelunga: A third party may not recover damages for intentional infliction of emotional distress when the defendant was not aware of the third party’s presence at the time that defendant’s conduct was taking place and the third party did not suffer any physical injury. One who, without a privilege to do so, causes severe emotional distress to another is liable (a) for such emotional distress and (b) for bodily harm resulting from it.

Elements of Intentional Infliction of Emotional Distress

1. The conduct must be extreme and outrageous a. Conduct must go beyond all bounds of decencyb. Conduct must be extreme and outrageous based on the reasonable person’s standardc. An individual’s sensitivity will not be taken into consideration unless the D is aware of

them.2. The conduct must be intentional or reckless

a. Purpose/ knowledgeb. Substantial certaintyc. Recklessness

3. There must be a casual connection between the wrongful conduct and the emotional distress. 4. Emotional Distress must be severe

e. P must show actual injuryf. There has to be a casual connection between the conduct and the emotional distress.g. Injury of P must not be physical.

C. Intentional Infliction of Emotional Distress on third parties 1. D must be aware of third party’s presence 2. Some jurisdictions require third parties to have suffered physical injuries as well.

6. Trespass to Land

Intentional entry upon the land of another without permission. In order to maintain an action for trespass to land, no physical injury to the land is necessary. The old common law rule was that all the space below and above the land was trepassable, now; however, it is limited to the space above and below the land of which the possessor could make reasonable use. If the intrusion interferes with the right to exclusive possession of property, the law of trespass applies.

1. Entry of an individual2. Causing the entry of another individual or an object or3. Tresspass may occur when one exceeds the scope of permission when staying on another’s land

through:a. exceeding the prescribed period of permission either because permission has been terminated

or the purpose for which it was allowed has been accomplished; orb. exceeding the purpose for which one is permitted to stay on the land; orc. going beyond the physical scope.

Dougherty v. Stepp: In order to maintain an action in trespass, it is not necessary that there be actual physical injury to the land. Trespass quare clausum fergit is the type of trespass whereby the trespasser “broke the close,” entered upon the land of the P.

Bradley v. American Smelting and Refining Co.: A landowner/possessor must show actual physical damages for recovery in trespass by intangible objects or substances.

Herrin v. Sutherland: It is trespass to pass over, or cause an object to pass over, the land of another, even of there has been no touching of the soil.

Rogers v. Board of Road Com’rs for Kent County: Trespass may be committed by the continued presence of a structure on land after the landowner has effectively terminated his consent to have the property on his land.

Page 6: Torts Rule Statements

§ 160 Failure to remove a thing placed on the land pursuant to a license or other privilegeA trespass, actionable under the rule stated in § 158, may be committed by the continued presence of the land of a structure, chattel or other thing which the actor or his predecessor in legal interest therein has placed thereon(a) with consent of the person then in possession of the land, if the actor fails to remove it after the consent has been effectively terminated, or(b) pursuant to a privilege conferred on the actor irrespective of the possessor’s consent, if the actor fails to remove it after the privilege has been terminated, by the accomplishment of its purpose or otherwise.

Elements of trespass of landVoluntary act

entry upon the land of another; orcausing another individual or an object to enter the land of another.

Intent1. Purpose/ knowledge to enter upon the land; or2. Substantial certainty that one will enter upon the land3. So long as one has the intent to enter, it is not necessary that one have intent upon the

land of another. Damages

1. No actual damage are required for physical entry2. Actual damages required in trespass by intangibles such as gases.

The entry must be unauthorized.

7. Trespass to Chattels

Intentional interference with a rightful owner’s/possessor’s use or possession of chattel. Unlike trespass to land, trespass to chattels is not actionable unless there is an actual injury suffered by the owner. The interference can be any physical contact with the chattel in a quantifiable way, or any dispossession of the chattel (whether by taking it, destroying it, or barring the owner's access to it). As opposed to the greater wrong of conversion, trespass to chattels is only actionable if actual damage can be shown.

One who commits a trespass to a chattel is subject to liability to the possessor of the chattel if, but only if: (a) he dispossesses the other of the chattel, or (b) the chattel is impaired as to its condition, quality, or value, or (c) the possessor is deprived of the use of the chattel for a substantial time, or (d) bodily harm is caused to the possessor, or harm is caused to some person or thing in which the possessor has a legally protected interest.

In sum, the basic elements of a claim of trespass to chattels are: 1) the lack of the plaintiff’s consent to the trespass, 2) interference or intermeddling with possessory interest, and 3) the intentionality of the defendant’s actions.

The measure of the damage in trespass to chattels is the actual diminution in the value of the chattel caused by the interference of the chattel.

Glidden v. Szybiak: An individual may not maintain an action for trespass to chattels unless he has suffered actual damages as a result of the trespass. (little girl got bitten by dog she was touching)

CompuServe Inc. v. Cyber Promotions Inc.: Trespass to chattel is actionable where the value or condition of the chattel is intentionally impaired. (Promotions company sending unsolicited email advertisements to Compuserve customers).

8. Conversion

Substantial interference with, or control and dominion over one’s possessory interest in chattel. An intentional exercise of dominion or control over a chattel, which so seriously interferes with the right of

Page 7: Torts Rule Statements

another to control it that the actor may justly be required to pay the other the full value of the chattel. The tortuous deprivation of another’s property without his/her authorization or justification; requires a wrongful taking, a wrongful detention, an illegal assumption of ownership, or an illegal use of another’s property.

1. Interference is so substantial that requires payment for the full value of chattel.2. Loss is more permanent than trespass to chattel.

Pearson v. Dodd: Photocopying of documents, which contain information not protected by property laws, does not constitute conversion.

Factors in providing conversion: (1) duration of dominion and control, (2) intent of the D, (3) D’s good faith, (4) extent of the resulting interference, (5) extent of the harm to chattel and (6) inconvenience and expense to owner.

Ways to convert the chattel: (1) stealing, (2) damaging, (3) using chattel (especially by a bailee- a person who holds the personal property of another for a specific purpose agreed to between the parties), (4) obtaining possession from a thief, (5) wrongfully selling chattel, (6) delivery of chattel to the wrong person, (7) refusing to return chattel to the rightful owner. R2 § 222A

What is the difference between trespass to chattels and conversion?

Both trespass to chattel and conversion are intended to protect the wrongful, intentional interference of personal property. The difference, however, is the degree of possession the interfering person has assumed. If the interfering person has merely challenged the right of possession, then there has been a trespass to chattel. If the interfering person, however, has created a virtual "forced sale" of the property to himself, then there is conversion.

For example, if someone steals property from you and you are able to recover the property with minimal or no damages, then you may have a cause of action in trespass to chattel. If, however, the same person steals the property and sells it to another, then you will have a cause of action in conversion. 

CH.3 Privileges

1. Consent

D who has committed an intentional tort may prevent liability by proving that the P consented to the D’s conduct. P consent must be knowing and voluntary. Consent obtained through fraud is ineffective. It is also ineffective if P is mistaken about a material fact, and D knows of the P’s mistake. Consent induced by fraud or misrepresentation as to a collateral matter, rather than fraud as to the essential character of the act itself, will not invalidate consent. R2 §§ 55, 57

Types of consent:1. Express: verbal or written contractual agreement occurs.2. Implied: Implied through P’s conduct, applying the reasonable prudent person standard: would a

reasonable prudent person in D’s position believe the P consented.

Medical care providers may act in the absence of express consent if (1) the patient is unable to give consent (unconscious, intoxicated, mentally ill, incompetent) (2) there is a risk of serious bodily harm if treatment is delayed, (3) a reasonable person would consent to treatment under the circumstances, and (4) the physician has no reason to believe this patient would refuse treatment under the circumstances.

Scope of consentD must stay within the scope of consent. Scope of consent may be determined based on custom and circumstances of the case.

Page 8: Torts Rule Statements

Consent to criminal actsIn the majority of jurisdictions, consent to criminal acts is ineffective. In a minority of jurisdictions, consent to criminal act is effective even of it involves a breach of peace.

O’Brien v. Cunard S.S. Co.: Where consent is not expressed, an individual may rely on the other party’s behavior and overt acts in order to determine whether that party has consented to the individual’s conduct.

Hackbart v. Cincinnati Bengals, Inc.: An individual is liable for the intentional striking of another during a violent game, where the rules of the sport prohibit such intentional acts.

Mohr v. Williams: A doctor, who has obtained the patient’s consent for a specific operation, may not perform another operation on the patient without his/her consent.

De May v. Roberts: An individual’s consent is not valid if the individual would not have consented, but for a mistaken belief about a material aspect of the conduct or transaction in question.

2. Self-Defense

Every individual is privileged to use reasonable force to defend against threatened battery by another. Amount of force allowed under self-defense is that which is reasonably necessary to protect against harm, which includes force which may cause serious bodily harm or death.

1. Good faith belief: individual must reasonably believe that use of force is necessary to protect against threat of harm.

2. Mistake does not vitiate (impair) the defense unless it is unreasonable. Threat of harm must be imminent

1. Harm must be occurring or about to occur.2. No self defense is allowed for battery or harm that has occurred in the past.3. Once the aggressor has retreated and the threat of harm is no longer imminent, the privilege

terminates. (Self-defense does not allow for retaliation).Provocations

1. Verbal threats alone do not justify the use of self-defense. 2. Provocations must be accompanied by the use of physical force.

Retreat1. Minority: before using force the party must retreat if he/she can do so.2. Majority: An individual does not have the duty to retreat.

3. Defense of Others

A person may use reasonable force to protect others against imminent threat of harm. Effect of mistake: (1) Majority- privilege only applies where the third party himself would be allowed to use self defense. If the intervener is mistaken, he or she is liable for battery even if the mistake was reasonable, (2) Minority- privilege to defend a third person applies so long as the intervener reasonably believes that force is necessary. Reasonable mistakes do not vitiate the privilege.

4. Defense of Property

A person is privileged to use reasonable force to protect personal or real property. Reasonable force (only as much force as appears necessary to protect the property) does not include deadly force or force which causes serious bodily harm. One may only use deadly force to (1) protect one’s own personal safety or that of other’s on the property, (2) prevent the commission of a crime, or (3) in some states if D gives P clear notice of danger.

Katko v. Briney: An owner of property may not protect his property from trespassers by use of direct or indirect force which may cause serious bodily injury or death.

Page 9: Torts Rule Statements

5. Recovery of Property

One may use reasonable force to recover chattel, which has unlawfully been take from him/her. Reasonable force does not include use of deadly force for recovery of property. Privilege may be used only if the original possessor’s possession of the property is not interrupted: (1) when the wrongdoer is in the process of taking the chattel or, (2) shortly after the wrongdoer has taken the property and the rightful owner/possessor is in hot pursuit of the wrongdoer.

Shopkeeper’s Privilege: merchants have the right to detain, for reasonable investigation, an individual whom they believe to have taken a chattel unlawfully. Elements: (1) reasonable belief that detainee has taken property and (2) reasonable time and manner of investigation.

Hodgeden v. Hubbard: A person may use reasonable force to recover property which is fraudulently obtained from him.

Bonkowski v. Arlan’s Department Store: A merchant, who reasonably believes that an individual has stolen merchandise from his/her store, may temporarily detain the individual for a reasonable investigation of the facts.

6. Necessity

In tort common law, the defense of necessity gives the state or an individual a privilege to take or use the property of another. A defendant typically invokes the defense of necessity only against the intentional torts of trespass to chattels, trespass to land, or conversion. A court will grant this privilege to a trespasser when the risk of harm to an individual or society is apparently and reasonably greater than the harm to the property. Unlike the privilege of self-defense, those who are harmed by individuals invoking the necessity privilege are usually free from any wrongdoing.

Public necessity is the use of private property by a public official for a public reason. The potential harm to society necessitates the destruction or use of private property for the greater good. The injured, private individual does not always recover for the damage caused by the necessity.

Surocco v. Geary: An individual who destroys another’s property in good faith, and under apparent necessity of preventing further harm to the community, os not personally liable in an action by the owner of the property destroyed.

Private necessity arises when an individual injures or damages the property of another in order to prevent injury to himself or to his own property or that of a third party because in that way a bigger harm will be prevented. Although the law allows individuals this privilege, unlike public necessity the individual who damages the property of another is liable for the damage caused. However, the individual is not liable for technical trespass.

Vincent v. Lake Erie Transp. Co.: An individual who damages the property of another as a result of private necessity is liable for the damage to that individual’s property.

7. Authority of Law

One who commits an action authorized by law is not liable in intentional torts. The difficulty with this privilege is to define the scope of the conduct that is within this privilege and, to determine whether the conduct of the defendant in question was in fact one that was authorized by law. Arrest falls within authority of law, either by citizen of police officer without a warrant, for breach of peace or felony committed in their

Page 10: Torts Rule Statements

presence. Individual can also arrest if they have a reasonable suspicion that the person committed the crime. A regular citizen, unlike a police officer, has the privilege ONLY if the felony was in fact committed.

8. Discipline

There are a number of relations which the necessity of some orderly discipline gives person who have the control of others the privilege of exercising reasonable force and restraint upon them without liability. (i.e. parent-child, teacher-student, etc…)

9. Justification

Justification can be used as a defense where a defendant’s conduct does not fall squarely within any other defense. Under this defense, the circumstances surrounding the defendant’s conduct will be looked at and assessed in order to determine the D acted reasonably in light of the circumstances. Also, the court could look at D’s position of authority and alternatives the D could have chosen in determining the reasonableness of his actions.

Sindle v. New York City Transit Authroity: Restraint or detention which is reasonable under the circumstances, imposed for the purpose of preventing another from inflicting personal injuries or property damage is not unlawful.

CH. 4 Negligence

The tort of negligence occurs when D’s conduct imposes an unreasonable risk of harm on the P, and the P has been injured as a result.

1. Elements of Cause of Action

1. A DUTY to use reasonable care. This is an obligation recognized by the law, requiring the actor to conform to a certain standard of conduct, for the protection of others against unreasonable risks.

2. A BREACH of duty is a failure to conform to the required standard. (Acted against his duty)3. CAUSATION- A reasonably close casual connection between the conduct and the resulting injury.

Involves two elements: (1) causation in fact and (2) proximate cause.4. Actual loss or DAMAGE resulting to the interest of another.

2. A Negligence Formula

An individual cannot be held liable for risks that are not reasonably known to him. An individual is not liable if his conduct conforms to the standard of care followed by the reasonably prudent person: (1) Where a person’s conduct fails to conform to the reasonableness standard, he may be liable even if the risk of harm is highly unlikely, (2) where a person uses a socially beneficial, yet inherently dangerous instrument, he must take reasonable precautions so the risk of harm does not outweigh the utility of the instrument, (3) where the burden of certain conduct, outweighs the risks, no liability attaches. RISKS MUST BE EVALUATED AS VIEWED AT THE TIME AT WHICH THEY OCCURRED.

§ 291 Unreasonableness; How Determined; Magnitude of Risk and Utility of Conduct

Where an act is one in which a reasonable man would recognize as involving a risk of harm to another, the risk is unreasonable and the act is negligent if the risk is of such magnitude as to outweigh what the law regards as the utility of the act or of the particular manner in which it is done.

Lubitz v. Wells: An individual cannot be held liable in negligence for risks that are not reasonably known to him. Although some objects like a gun are inherently dangerous that it is negligent to leave them lying around without special handling, other objects like golf clubs pose less of a danger and it

Page 11: Torts Rule Statements

is not negligent to leave them around even if it turns out that, unexpectedly, they caused harm. The risk is to be evaluated as it reasonably seemed before the accident.

Blyth v. Birmingham Waterworks: An individual is not negligent if his/her conduct and precautions conform to the standard followed by a prudent or reasonable person.

Gulf Refining Co. v. Williams: An individual may be liable in negligence even if his/her conduct creates a risk of harm which is highly improbable and unlikely.

Chicago, B. & Q.R. Co. v. Krayenbuhl: An individual using a socially beneficial yet inherently dangerous instrument, must take reasonable precautions such that the risk of injury posed by the instrument does not outweigh its utility.

Davison v. Snohomish County: A county is not negligent for falling to build safety railing where the construction of such railing would create an undue burden on the county and prevent construction of other roads.

United States v. Carroll Towing Company: Learned Hand Balancing Test: If P (probability of the problem arising) x L (the gravity of resulting injury) > B (the burden of adequate precautions) = liability exists. Meaning, the degree of care demanded of a person by an occasion is the resultant of three factors: the likelihood that the conduct will injure others, taken with the seriousness of the possible injury, and balanced with the interest which he must sacrifice to avoid the injury. Basically, if the likelihood of harm and the injury that harm can cause are greater than the costs of precaution, those precautions must be taken. An individual may be liable for failing to take reasonable precaution against great risk of injury even where the probability of injury occurring is very little.

3. The Standard of Care

Our negligence system focuses on the actor’s level of care in carrying out the activity, a defendant who engages in a fairly safe activity but does so negligently is likely to be liable for damages.

(A) The Reasonable Prudent Person

Objective standard: What would the average reasonable person of ordinary prudence under similar circumstances? “Would a reasonable person of ordinary prudence, in the position of the defendant, have conducted himself as the defendant did?”

Vaughan v. Menlove: The standard of care used to determine negligence is that of the reasonable prudent person in similar circumstances and not the good faith or subjective standard adhered to by the D. (fire started by hay rick)

Delair v. McAdoo: A reasonable driver of an automobile should be charged with the knowledge that driving with warn tires poses a danger to the other individuals on the road.

Trimarco v. Klein: Evidence of custom, common usage, and practice, although compelling, may not be used as the compelling test of negligence.

Cordas v. Peerless Transportation Co.: The conduct of an individual who is in an emergency situation is not measured by the standard of care of a reasonable person in a non-emergency situation. The law does not hold one in an emergency to the exercise of mature judgment required of him under circumstances where he has an opportunity for deliberate action.

Roberts v. State of Louisiana: In determining the negligence of an individual, the court must take into consideration the physical attributes or shortcomings of the individual to determine whether an individual has taken reasonable care in his/her conduct. In other words, we have to ask ourselves how an average, reasonable, person with the same handicap, would conduct himself in the same circumstance.

Robinson v. Lindsay: A minor child engaged in the operation of machinery ordinarily used by adults is held to an adult standard of care.

Breunig v. American Family Ins. Co.: Insanity is not a defense to negligence conduct where the defendant had prior warning and knowledge of his/her insanity.

(B) The Professional

Page 12: Torts Rule Statements

Professionals are usually held to a higher standard of care, the standard of care of the ordinary prudent person with skill and knowledge that is commonly held by professionals in the community.

Heath v. Swift Wings, Inc.: A professional’s negligence is not determined based on the degree of ordinary care and caution exercised by professionals having the same experience and training.

Hodges v. Carter: An attorney who has acted in good faith and in the best interests of his client is not liable in negligence for mere error in judgement.

Boyce v. Brown: In a medical malpractice case, the plaintiff must prove the standard of care in the medical community and that the defendant’s conduct departed from this standard.

Morrison v. MacNamara: The conduct of health professionals who are certified through a system of national board certification must be measured against nationally accepted standards of care, as opposed to local standards of care.

Scott v. Bradford: A physician is liable in negligence if he does not inform the patient of his options and its attendant risks before treating the patient. If treatment is completely unauthorized and performed without any consent at all, there has been a battery. However, if the physician obtains a patient’s consent but has breached his duty to inform, the patient has a cause of action sounding in negligence for failure to inform the patient of his options, regardless of the due care exercised at treatment, assuming there is injury. For medical malpractice actions: patient must show: (1) D failed to inform him adequately of a material risk before securing his consent to the proposed treatment, (2) if he had been informed of the risks he would not have consented to the treatment, (3) the adverse consequences that were not made known did in fact occur and he was injured as a result of submitting to the treatment.

Moore v. The Regents of the University of California: In order to satisfy his fiduciary duty to the patient and obtain the patient’s consent, a physician who is seeking a patients consent for a medical procedure must disclose his own personal interests (i.e. research or economic), unrelated to the patient’s health, that may affect the patient’s decision. Failure to disclose such interests may give rise to a cause of action for performing medical procedures without informed consent or breach of fiduciary duty.

Note: The professional contracts to render services, but the suit is usually in the tort for damage caused by negligence. There is no need to contract specifically to exercise the normal skill of the professional- the law imposes the duty.

(C) Aggravated Negligence

Degrees of care: The care required by the standard of the reasonable person varies according to the risk. As the danger increases, the actor is required to the exercise caution proportionate to it. Those who deal with things that are more dangerous, such as explosives or electricity, must exercise more care than one who is walking down the street. Likewise, those who have accepted a special responsibility towards others, must exercise more care in accordance with the duty undertaken.

Degrees of Negligence: Slight negligence- failure to use great care, ordinary care- failure to use reasonable care, and gross negligence- failure to use even slight care. These have largely lost their significance though.

Willful, wanton, reckless conduct: This type of conduct is defined as consisting of a deliberate and conscious disregard for a known high degree of probability of harm to another.

4. Rules of Law

Negligence standards of care are not ordinarily determinate as a rule of law. Rather, the standards are subjective, based on the exigencies of each situation. In Baltimore & Ohio R. Co. v. Goodman, Justice Holmes concluded that one standard had to be set out to be used by all courts, but his rule was too strict and inflexible to be used in all cases. He has said that if a driver wanted to cross over railroad tracks, he

Page 13: Torts Rule Statements

must first stop his vehicle, look down the track and if necessary get out of his vehicle for an additional look. In Pakora, however, the court decided that was too inflexible.

Pokora v. Wasbash Ry. Co.: An automobile driver who is crossing over the railroad tracks does not, under all circumstances, have the absolute legal duty to stop the car and look for oncoming trains.

5. Violation of Statute

Sometimes the legislature passes a statute that appears to define reasonable conduct in a certain kind of situation. A court may treat the violation of such legislation as negligence per se. The negligence per se doctrine is the general rule that when a safety statute has a sufficiently close application to the facts of the case at hand, an unexcused violation of that statute is negligence per se, and the D will not be allowed to show that the legislature set an unduly high standard of care.

Three requirements for application of negligence per se:1. D violated a Statute2. the statute was designed to protect against the same type of accident that D’s conduct caused; and3. the accident victim falls within the class of persons that the statute was designed to protect.

Osborne v. McMasters: An individual who breaches a duty imposed on him by statute is liable for injuries to those whom the statute is designed to protect if the injuries resulted from the breach. Where a statute or ordinance imposes upon any person a specific duty to for the protection of others, if he neglects to perform the duty he is liable to those whose protection or benefit it has imposed for any injury, when caused proximately.

(A) Applicability of Statute

Stachniewicz v. Mar-Cam Corp.: A violation of a statute or regulation constitutes negligence per se when the violation results in injury to a member of the class of persons protected by the statute or regulation and when the harm is of the kind which the statute or regulation is designed to prevent.

Ney v. Yellow Cab Co.: A plaintiff who bases his claim for negligence on a defendant’s violation of a statute, must prove that his injuries were directly and proximately caused by the defendant’s violation of the statute.

IMPORTANT TERMS: - Intervening cause: Also known as “supervening,” that cause which comes into active

operation in producing the result after the negligence of the defendant. A separate act or omission that breaks the direct connection between the defendant’s actions and an injury or loss to another person, and may relieve the defendant of liability for the injury or loss.

- Proximate cause: The type of cause which in natural and continuous sequence unbroken by any new independent cause produces an event, and without which the injury would not have occurred. It is an event sufficiently related to a legally recognizable injury to be held the cause of that injury.

Perry v. S.N. and S.N.: A child abuse reporting statute does not create civil liability for one who fails to report abuse.

Note: The court must decide whether a criminal statute can be used as a standard of care in a tort suit. In doing so, the court determines the same two things as in negligence per se.

(B) Effect of Statute

Martin v. Herzog: Proof of negligent conduct, without a showing of casual connection between the conduct and injury suffered, is not enough to impose liability on the party whose conduct was

Page 14: Torts Rule Statements

negligent. One who violates a statute imposing a standard of care has fallen below that standard, and therefore is negligent. However, for this negligence to be actionable a P who has suffered an injury because of the violation must prove that his/her injury was caused by the violation.

Zeni v. Anderson: The violation of a statute which is found to apply to a particular set of facts creates a presumption which can be rebutted by a showing of an adequate excuse or justification for a violation of the statute. Possible excuses: violation is reasonable because of the actor’s incapacity, neither knows nor should know of the occasion for compliance, he is unable after reasonable diligence or care to comply, confronted by an emergency not due to his own misconduct, compliance would involve a greater risk of harm to the actor or others. R2 § 288A.

6. Proof of Negligence

P has three burdens of proof on the issue of negligence- the burden of pleading, the burden of coming forward with enough evidence to avoid a directed verdict against him, and if he hurdles that, the burden of persuading the trier of fact to find in his favor.

(A) Court and Jury: Circumstantial Evidence

Circumstantial evidence has been defined as “evidence of one fact…from which the existence of the fact to be determined may reasonably be inferred. It involves in addition to the assertions of witnesses as to what they have observed, a process of reasoning, or inference, by which a conclusion is drawn.”

Function of judge and jury: Both the judge and jury play a significant role in the adjudication of a negligence case. The judge decides all questions of law, meaning, that the judge will decide whether reasonable people could differ as to the facts of the case, if the judge decides they could, he/she will make it a jury question. The judge will also determine what the D’s duty to the P was. Finally, the judge may remove the case from the jury by directing the verdict. The jury is the finder of facts. That means if reasonable persons may differ as to the facts the jury must determine (1) what happened and (2) the particular standard of care (whether the facts as found indicate that the defendant breached his duty of care to the P in a way that proximately caused the P’s injury).

Goddard v. Boston & Maine R.R. Co.: In a negligence action, in order to avoid a directed verdict, the plaintiff must provide sufficient evidence from which reasonable jurors could infer that the defendant was engaged in negligent conduct and that his negligence caused the plaintiff’s injury.

Anjou v. Boston Elevated Railway Co.: A directed verdict in favor of the defendant is not appropriate where the plaintiff has presented evidence sufficient to support an inference of negligence on the part of the defendant. The banana peel was dark in color and gritty in texture, therefore it could be inferred that the peel had been there for a while and not just dropped by another passenger and if the D’s employee had exercised reasonable care, they would have picked up the banana peel.

Joye v. Great Atlantic and Pacific Tea Co.: Actual or constructive knowledge of a dangerous condition, established through direct or circumstantial evidence, must be proven to impose liability for injuries caused by the condition.

Ortega v. Kmart Corp.: A storeowner owes its patrons a duty to exercise a reasonable care in keeping the premises reasonably safe. To impose liability for injuries suffered by an invitee due to a hazardous condition, the plaintiff must show that the defendant had either actual or constructive knowledge of the dangerous condition, or should have been able to discover the hazardous condition by the exercise of ordinary care. A hazardous condition is one that poses an unreasonable risk to invitees on the premises.

Jasko v. F.W. Woolworth Co.: A plaintiff need not prove defendant’s notice of the dangerous condition which has caused her injuries, when the dangerous condition is one which is continuous and easily foreseeable.

H.E. Butt Groc. Co. v. Resendez: In order to recover, the defendant needed to prove actual or constructive knowledge of a dangerous condition, that the condition posed an unreasonable risk of

Page 15: Torts Rule Statements

harm, that the plaintiff did not exercise reasonable care to mitigate the risk, and that the failure to use such care proximately caused injury.

(B) Res Ispa Loquitur

Res ipsa loquitur is a common law theory on the use of circumstantial evidence in tort liability on a negligence theory. The term comes from Latin and means "the thing itself speaks," but is more often translated "the thing speaks for itself." This theory allows the P to point to the fact of the accident and to create an inference that, even without a precise showing of how the D behaved, the D was probably negligent, it makes the P’s task significantly easier. The theory allows the plaintiff to use circumstantial evidence to meet the burden of proof in negligence cases for only the first two elements: duty and breach. Proving these two elements means that, in fact, the defendant was negligent. However, the plaintiff must still prove, by a preponderance of the evidence, that the defendant's negligence was the cause-in-fact of the harm and the proximate cause of the harm before he can recover from the defendant.

The Restatement (Third) of Torts, § 17 describes a two step process for establishing res ipsa loquitur. The first step is (1) whether the accident is the kind that would usually be caused by negligence, and the second is (2) whether or not the defendant had exclusive control over the instrumentality that caused the accident. If found, res ipsa loquitur creates an inference of negligence.

Byrne v. Boadle: A plaintiff need not provide evidence of negligence on the part of the D when the mere fact of the accident or injury having occurred is evidence of negligence of the D.

McDougald v. Perry: Res ispa loquitur provides an inference of negligence when the accident is the type that does not occur without a negligence and the defendant is in control of the circumstance.

Larson v. St. Francis Hotel: The doctrine of res ipsa loquitur does not apply in a case where the plaintiff’s injury could be attributable to more than one cause, some of which are not within the exclusive control and management of the defendant. A hotel has does not have exclusive control, either actual or potential, of its furniture. The guests have at least partial control.

Ybarra v. Spangard: Res ispa loquitur may apply to situations where more than one D is the cause of the P’s injury and the various instrumentalities which caused the plaintiff’s injuries were in control of the D in question.

Sullivan v. Crabtree: Where a P establishes the prima facie case for defendant’s negligence based on res ispa loquitur, the court must not always as a matter of law enter a verdict for the P. The truck reck could be due to (1) drivers negligence or (2) no fault of his own, thought an unavoidable accident cased by the breaks failing or some other mechanism in the trick failing to work properly. Since both inferences are possible from the information in this case, it is proper that a jury should decide the issue of negligence.

CH. 5 Causation in Fact.

1. Sine Qua Non

In order to prove that a defendant caused the plaintiff’s injury, the plaintiff must prove two separate aspects of causation. The first is known as causation in fact, also called “but for” causation or sine qua non. The plaintiff must show that the injury would not have occurred without the defendant’s negligent conduct. A defendant’s negligence may be the cause of an injury if it is a substantial factor in causing the harm. A mere suspicion that a party was the cause of an accident is insufficient. There must be some actual evidence that the party was negligent.

Page 16: Torts Rule Statements

Perkins v. Texas and New Orleans Ry. Co.: Negligence is the cause in fact of injury to another if it is a substantial factor in causing the harm. You cannot sue for negligence unless such negligence was an actual cause of the harm. Negligence is an actual cause if it was a substantial factor in bringing about the harm.

2. Proof of Causation

Where a defendant’s negligence greatly increases the chance of an injury, the possibility that the accident could have occurred without the negligence does not break the chain of causation. The P bears the burden of proving that the D actually caused the injury, just a she must bear the burden of proving the other parts of his prima facie case by preponderance of the evidence.

Reynolds v. Texas & Pac Ry. Co.: Where the negligence of a defendant greatly increases the chances of an accident giving rise to the plaintiff’s injury, the possibility that the accident could have happened without the negligence does not break the chain causation.

Gentry v. Douglas Hereford Ranch, Inc.: Evidence that defendant’s conduct was a cause in fact of the accident must be offered to make a negligence case.

Kramer Service, Inc. v. Wilkins: In order to impose liability on a D, it is not sufficient that the P prove that the D’s negligence and the P’s injury co-existed or that the injury came after the negligent conduct of the D.

Herskovits v. Group Health Cooperative of Puget Sound: An action in a negligence can be maintained where probable reduction in the decedent’s chance for survival can be proven, even though the decedent would not have lived to normal life expectancy.

Daubert v. Merrell Dow Pharmaceuticals, Inc.: The Federal Rules of Evidence do not require “general acceptance” of a scientific technique in the scientific community as an absolute prerequisite to admissibility of expert witness testimony.

3. Concurrent Causes

Where two separate acts of negligence combine to cause an injury to a third party, each actor is liable, even though the injury would not have happened but for the negligence of the other actor. Inclusive as the “but for” test is, it nonetheless rules out one kind of cause which the courts have generally held does constitute a true cause in fact. This is the situation where two events concur to cause harm, and either one would have been sufficient to cause substantially the same harm without the other. To provide for this case, it is generally stated that each of these concurring events is a cause of the injury, insofar as it would have been sufficient to bring that injury about.

Substantial factor standard: Where each of the two events would have been sufficient by itself to bring about the harm, the test for each event is often said to be whether it was a substantial factor in bringing about the harm. If so, that harm is a cause in fact.

Hill v. Edmonds: Where separate acts of negligence combine to produce a single injury, each tortfeasor is liable for the entire result even though his act alone may not have caused the result.

Anderson v. Minneapolis, St. P. & S. St. M. R.R. Co.: Where a plaintiff is injured by the negligent conduct of more than one tortfeasor, each is independently liable if it was a substantial factor in bringing about the plaintiff’s injury.

4. Problems in Determining which Party Caused the Harm

Summers v. Tice: Where defendants are negligent toward the plaintiff, but only one defendant’s negligence has caused the plaintiff’s injury, the plaintiff may recover from both defendants even though the plaintiff is unable to show which defendant actually caused the plaintiff’s injury. In that case, each defendant has the burden to proof he did not cause the injury.

Page 17: Torts Rule Statements

Sindell v. Abbott Laboratories: Where more than two defendants have negligently produced a drug causing the plaintiff’s injury, the latter may recover from the defendants even though the plaintiff does not prove which defendant’s drug was the direct cause of the plaintiff’s injury. The market share theory: If P cannot prove which of the three or more persons caused the injury, but can show all were negligent (or produced a defective product), the court may cast upon each D the burden to proving that he did not cause the injury. If a given member of the class of D’s is unable to prove that he did not cause the injury, the court may well require him to pay that percentage of the P’s injury which the D’s sales of the product bore to the total market sales of that type of product.

CH. 6 Proximate or Legal Cause

In addition to proving causation in fact, a plaintiff must show that the defendant’s conduct was the proximate cause of the injury. Proximate cause requires a natural sequence, unbroken by intervening causes, that results in some injury. A proximate cause is an event sufficiently related to a legally recognizable injury to be held the cause of that injury.

Atlantic Coast Line R. Co. v. Daniels: Courts should not set rigid limitations in defining proximate cause as a basis of liability.

1. Unforeseeable Consequences

A defendant is liable only for the ordinary and natural results of his negligent conduct. The foreseeability of the consequences of a defendant’s actions depend on the balancing between the likelihood of risk and the magnitude of damages flowing therefrom.

Ryan v. New York Central R.R. Co.: A defendant is only liable for the ordinary and natural results of his negligent conduct. The action cannot be sustained, for the reason that the damages incurred are not the immediate but the remote result of the negligence of the Ds. The immediate result was the destruction of their own wood and sheds, beyond that, it was remote.

Bartolone v. Jackovich: A D must take a P as he finds him and is liable to the plaintiff in damages for the aggravation of plaintiff’s pre-existing illness due to the defendant’s negligent conduct.

In re Arbitration Between Polemis and Furness, Withy & Co., Ltd.: Where it is reasonably foreseeable that the defendant’s negligent conduct would cause damages to the plaintiff, the defendant is liable even though the exact extent of the damages is not foreseeable. If the act would or might probably cause the damage, the fact that the damage it in fact causes is not the exact kind of damage one would expect, is immaterial, so long as the damage is in fact directly traceable to the negligent act, and not due to the operation of independent causes having no connection with the negligent act, except that they could not avoid its results.

Overseas Tankship (U.K.) Ltd. v. Morts Dock & Engineering Co., Ltd. “Wagon Mound No. 1”: A defendant is not liable for unforeseeable consequences of his negligent conduct, even though they were the direct result of defendant’s conduct. “It does not seem consonant with our current ideas of justice or morality that for an act of negligence, however slight or venial, which results in some trivial unforeseeable damage the actor should be liable for all consequences, however unforeseeable and however grave, so long as they can be said to be direct.”

Overseas Tankship (U.K.) Ltd. v. Miller Steamship Co. “Wagon Mound No. 2”: The foreseeability of the consequences of a defendant’s actions depend on the balancing between the likelihood of risk and the magnitude of damages flowing therefrom.

Palsgraf v. Long Island R.R. Co.: (Cardozo) A defendant owes a duty of care only to those plaintiffs who are in the reasonably foreseeable zone of danger. ZONE OF FORESEEABILITY (Andrews,

Page 18: Torts Rule Statements

dissenting) Where an act threatens the safety of others, the doer is liable for all its proximate consequences, even when the injury is to one who would generally be thought to be outside the radius of danger. UNBROKEN, NATURAL SEQUENCE OF EVENTS

Yun v. Ford Motor Co.: When the court determines that an injury was not reasonably, foreseeably caused by the defendant, the issue of proximate cause may be taken away from the jury. Where concurrent forces are involve, the manufacturer of a defective product may negate strict liability upon a showing of an intervening, superseding cause or the existence of another sole proximate cause of his own injuries.

To test for forseeability:

1. Ask - what was the negligence? In Palsgraff, the negligence was the LIRR employee pushing the passenger and causing him to drop the box.

2. Ask - what was the risk created from that negligence? In Palsgraff, the risk from pushing the passenger was that the passenger himself could have been hurt if the push was really hard or that the employee's push could cause damage to the contents of the box that the passenger was holding.

3. Ask - was the injury that occurred a result of the risk that was created from that negligence? In Palsgraff, the harm was not within the scope of the risk (see above). Thus, it was not foreseeable. Pushing someone on to a train with a package in their hand does not create a risk that a bystander will be hurt by an explosion. LIRR only would have been liable for damage to the passenger who was pushed on the train and possibly for the contents of the package that was dropped.

That, however, would not be the same result if Polemis was used- the direct causation (hindsight) view holds that the D is liable for all consequences of her negligent act, provided that these consequences are not due in part to a superseding intervening cause (Andrews view was nebulous test).

2. Intervening Causes

A separate act or omission that breaks the direct connection between the defendant's actions and an injury or loss to another person, and may relieve the defendant of liability for the injury or loss. An intervening cause relieves a defendant of liability only if it would not have been foreseeable to a reasonable person, and only if damage resulting from the defendant's own actions would not have been foreseeable to a reasonable person. Some jurisdictions use two terms to define the intervening cause doctrine: intervening cause and superseding cause. In these jurisdictions intervening cause describes any cause that comes between a defendant's conduct and the resulting injury, and an intervening cause that relieves a defendant of liability is called a superseding cause. Other jurisdictions do not use the term superseding cause. These jurisdictions simply ask whether the intervening cause is sufficient to relieve a defendant of liability. All jurisdictions differentiate between an intervening cause that relieves a defendant of liability and one that does not: the only difference is in the terminology.

Derdiarian v. Felix Contracting Corp.: Where the act of a third party intervenes between the defendant’s negligent conduct and the plaintiff’s injury, the casual connection is not severed if the intervening act is a normal and foreseeable consequences of the risk created by the defendant’s negligent conduct. An intervening act may not serve as a superseding cause, and relieve an actor of responsibility where the risk of the intervening act occurring is the very same risk which renders the acto negligent.

Watson v. Kentucky & Indiana Bridge & R.R. Co.: A negligent defendant is not liable for the intentional intervening malicious acts of a third party which are not reasonably foreseeable. The mere fact that the concurrent cause or intervening act was unforeseen will not relieve the D guilty of

Page 19: Torts Rule Statements

the primary negligence from liability, but if the intervening agency is something so unexpected or extraordinary as that he could not or ought not to have anticipated it, he will not be liable, and certainly he is not bound to anticipate the criminal acts of others by which damage is inflicted and hence is not liable therefore.

Fuller v. Preis: An act of suicide is not, as a matter of law, a superseding cause in negligence law which precludes liability of the initial tortfeasor.

McCoy v. American Suzuki Motor Corp.: The rescue doctrine may be invoked in product liability actions and, if it is, the rescuer is required to show the defendant proximately caused his injuries. This means the rescuer is allowed to sue the party which caused the danger requiring the rescue in the first place, but the P must show the D proximately caused his injuries.

Essentially, in its pure form the Rescue Doctrine boils down to 4 main elements - all of which must be met in order to bring it to bear for the person asserting its privilege.

1. There must be peril or the appearance of peril to a third party, caused by the defendant. 2. That peril or appearance of peril must be imminent 3. A reasonable person would recognize the peril or appearance of peril and the plaintiff must also have

actually recognized it. 4. The plaintiff must have exercised reasonable care in effecting the rescue.

Note: One who fails in his duty to remedy a defective or dangerous condition is liable for the injuries resulting therefrom, although the immediate cause of the injury is an act of God.

Dependent vs. Independent intervening causes: A dependent intervening cause is set in motion by the D's own conduct, and will not relieve the D of liability unless it is extraordinary. For example, suppose the D poked an associate in the chest during a friendly discussion around a watercooler, and the associate subsequently jumped out a window. This unusual reaction may be deemed an extraordinary intervening cause that relieves the D of liability.An independent intervening cause arises through no fault of the defendant. It relieves a D of liability unless it was foreseeable by the defendant.

3. Public Policy

Kelly v. Gwinnell: A host is liable for the negligence of an adult social guest who has become visibly intoxicated at the host’s home, and where risk of harm to others is foreseeable. (The great weight of authority supports the view that in the case of an ordinary able-bodied man, it is the consumption of the alcohol, rather than the furnishing of the alcohol which the proximate cause of any subsequent occurrence.)

Enright v. Eli Lilly & Co.: An injury to a mother, which results in injuries to later conceived child, does not establish a cause of action in favor of the child against the original tortfeasors.

4. Shifting Responsibility

We have encountered several situations where the D acts negligently, and his negligence or other wrongdoing of third persons, in combination produces the P’s injury. Those are situations in which the D will be allowed to say, in effect, that responsibility for the dangerous conditions created in part by her has passed to that third person, absolving the D of responsibility. That is, the responsibility is to be “shifted.”

Difference between cause in fact and proximate cause: cause in fact is what actually caused the injury (i.e. in Palsgraf, it would have been the package exploding). However, proximate cause looks at the natural sequence of events or zone of foresseability (whichever the jurisdiction uses). The distinction between cause in fact and proximate or legal cause is not merely on exercise in semantics. The terms are not

Page 20: Torts Rule Statements

interchangeable. Cause in fact refers to the cause and effect relationship between the D’s tortuous conduct and the P’s injury or loss. Thus cause in fact deals with the but for consequences of an act. The D’s conduct is a cause of the event if the event would not have occurred but for that conduct. In contrast, proximate cause or legal cause concerns a determination of whether legal liability should be imposed where the cause in fact has been established. Proximate or legal cause is a policy decision made by the legislature or the courts to deny liability for otherwise actionable conduct based on considerations of logic, common sense, policy, precedent and ‘our more or less expressed ideas of what justice demands or of what is administratively possible or convenient.’

CH. 7 Joint Tortfeasors

Joint tortfeasors are two or more persons whose negligence in a single accident or event causes damages to another person. In many cases the joint tortfeasors are jointly and severally liable for the damages, meaning that any of them can be responsible to pay the entire amount, no matter how unequal the negligence of each party was. Under several liability, each tortfeasor pays no more than his apportioned share, and the injured party bears the loss of any uncollectible share. Some courts have found that the public policy reasons for requiring tortfeasors rather than injured persons to bear the burden of the uncollectible portions of damages continue to control even with the adoption of comparative negligence. If more than one person is a proximate cause of the P’s harm, the harm is indivisible, then under the traditional common-law rule, each defendant is liable for the entire harm. The liability is said to be joint and several. (Meaning the P can sue and collect from either of them or both of them.)

Three types of factual situations that create joint and several liability: (1) tortfeasor acted in concert, (2) D’s failed to perform a common duty to the P, (3) D’s who acted independently to cause an indivisible harm.

1. Liability and Joinder of Defendants

Joinder is the joining of parties as defendants in a suit, it is permitted where the D’s acted in concert, when the D’s acted independently to cause the same harm and even when the D’s acted together to cause different harms.

Bierczynski v. Rogers: Two parties who engage in an illegal car race are wrongdoers acting in concert and thus are jointly liable for injuries to a third person regardless of which party directly inflicted the injury or damage upon the third person.

Coney v. J.L.G. Industries, Inc.: The doctrine of comparative negligence does not eliminate joint and several liability. Comparative negligence does apply to product liability torts. "[If the court were to] eliminate joint and several liability as the defendant advocates, the burden of the insolvent or immune defendant would fall on the plaintiff; in that circumstance, plaintiff's damages would be reduced beyond the percentage of fault attributable to him," and thus joint and several liability remains under comparative negligence.

Bartlett v. New Mexico Welding Supply, Inc.: In a comparative negligence jurisdiction, a concurrent tortfeasor is not jointly and severally liable for the entire amount of the plaintiff’s judgment, they are only liable for their percentage of culpability in terms of damages.

2. Satisfaction and Release

A P may bring a series of separate actions against D’s liable for the same damage and take each to judgment so long as he only collects on one. There can be only one full satisfaction, if there is partial satisfaction, it must be credited to the other parties who are also liable. Satisfaction of judgment is paying off the sum ordered in a court judgment or fulfillment of all obligation under it.

A P who has a possible cause of action against two or more D’s, may settle with one while pursuing a law suit against the remainder. Until recently, settling with one, meant releasing the other D’s as well. This rule was the product of the common law fiction that a P had only one, indivisible cause of action against all joint

Page 21: Torts Rule Statements

tortfeasors, and it could not be extinguished as to one and alive as to the others. Release is basically liberation from the obligation or duty to pay claim.

Bundt v. Embro: A party who has suffered an indivisible harm by negligence of several parties in not entitled to more than one satisfaction for the same harm.

Cox v. Pearl Investment Co.: A contract between one tortfeasor and the plaintiff, where the parties intend to reserve to the plaintiff the right to sue the co-tortfeasor, does not release the co-tortfeasor from liability.

Elbaor v. Smith: Mary Carter agreements (a K by which one or more but not all co-D’s settle with the P and obtain a release, along with a provision granting them a portion of any recovery from the nonparticipating co-D’s), where a settling defendant has a stake in the outcome of a case against co-defendants, are violative of public policy.

3. Contribution and Indemnity

Contribution is when the D get reimbursement against others who were also negligent with him and caused the harm.

Knell v. Feltman: The right to contribution does not exist only between tortfeasors against whom the plaintiff has obtained a judgment. The common law rule disallowing contribution between non-intentional tortfeasors is hereby overruled. Otherwise, the P and one D could gang up against another D and force him to pay for the P’s entire damages. The appellant’s theory that there can be no contribution unless the Pl has first obtained a judgment against both wrongdoers is baseless for another reason.  The right to seek contribution belongs to the tortfeasor who has been forced to pay, and the existence of the right cannot logically depend upon a selection of dfs made by the Pl.  When a tort is committed by the concurrent negligence of two or more persons who are not intentional wrongdoers, contribution should be enforced; that joint judgment against such tortfeasor is not a prerequisite to contribution between them , and it is immaterial whether they were, or any of them was, personally negligent.

Yellow Cab Co. of D.C., Inc. v. Dreslin: Joint liability is required before any contribution can be ordered. Contribution may not be obtained from a tortfeasor who has no liability to the injured party. (immunity in this case of husband nor wife is liable for the tortuous acts of one against the other.) Majority rule that non-immune tortfeasors may not seek contribution or indemnity from those who are immune.

Slocum v. Donahue: A joint tortfeasor is not entitled to indemnification from a fellow tortfeasor. Since Donahues released Ford, that at the same time discharges Ford from liability for contribution to Donahues. He is not entitled to indemnification either because indemnity allows someone is without fault, compelled by law to defend himself against the wrongful act of another, to recover from the wrongdoer the amount of his loss. Indemnity is limited to those cases where the would-be indemnity (Donahue) is derivatively or vicariously liable for the wrong act of another.

4. Apportionment of Damages

Bruckman v. Pena: A defendant cannot be held liable for a plaintiff’s subsequent injury where the plaintiff cannot apportion the damages between the causes of injuries.

Michie v. Great Lakes Steel Division, Nat’l Steel Corp.: Where the independent concurring acts of defendants have caused an indivisible harm to the plaintiff and no reasonable means of apportioning the damages is evident, the court may hold the defendant’s jointly and severally liable.

Dillon v. Twin State Gas & Electric Co.: Where a plaintiff, regardless of the defendant’s negligence, would have suffered injury, the plaintiff’s damages would be measured based on the plaintiff’s injured condition.

CH. 8 Duty of Care

Page 22: Torts Rule Statements

The proposition is that whenever one person is by circumstances placed in such a position with regard to another that every one of ordinary sense would at once recognize that if he did not use ordinary care and skill in his own conduct with regard to those circumstances, he would cause danger of injury to the person or property of the other, a duty arises to use ordinary care and skill to avoid such danger.

1. Privity of Contract

Nofeasance- D made a promise and broke it, has a contract action. Misfeasance- D attempted to perform, but did the wrong thing, recovery in tort is greatly augmented as well as breach of k.

Winterbottom v. Wright: A person may not sue another on a non-public contract where he/she is not privy (having mutual interests in the same action or thing) to the contract.

MacPherson v. Buick Motor Co.: A manufacturer owes a duty of care to remote purchasers if the product is reasonably certain to cause the injury when negligently made. If the nature of the thing is such that it is reasonablt certain to place life and limb in peril when negligently made, it is then a thing of danger. Its nature gives warning of the consequences to be expected. If to the element of danger there is added knowledge that the thing will be used by persons other than the purchaser, and used without new tests, then, irrespective of contract, the manufacturer of this thing of danger is under a duty to make it carefully. That is as far as we need to go for the decision of this case . . . . If he is negligent, where danger is to be foreseen, a liability will follow.

H.R. Moch Co. v. Rensselaer Water Co.: A party who is in no contractual or special relationship to another may not be held liable for refusal to aid the other.

Clagett v. Dacy: An attorney may not be held liable to a person who is not intended to benefit by his performance, and with whom the attorney does not have a contractual relationship.

2. Failure to Act

Generally, a person cannot be liable in tort for failure to act. However, there are exceptions to this rule including employer to employees, school with students, landlord with tenants, ect…

Hegel v. Langsam: A university does not have the duty to control the private lives of its students. L.S. Ayres & Co. v. Hicks: A store owner of an inviter has the duty to rescue a person in peril if the

instrumentality causing the person’s injury is in control of the owner or inviter. J.S. and M.S. v. R.T.H.: When a spouse has actual knowledge or special reason to know of the

likelihood of his or her spouse engaging in sexually abusive behavior against a particular person or persons, a spouse has a duty of care to take reasonable steps to prevent or warn of the harm and a breach of such duty constitutes a proximate cause of the resultant injury.

Tarasoff v. Regents of University of California: The relationship between a therapist and a patient supports the duty on the part of the therapist to exercise reasonable care to others against dangers posed by the plaintiff’s illness.

3. Pure Economic Loss

If the harm is pure economic loss, the courts take more seriously the claim that liability should be restricted. Cases fall into two categories: (1) Negligent misrepresentation or misstatement causing economic loss and (2) negligent acts causing economic loss.

State of Louisiana ex rel. Guste v. M/V Testbank: A plaintiff may not recover for pure economic loss if the loss resulted from physical damage to the property in which the plaintiff had no interest.

4. Emotional Distress

Page 23: Torts Rule Statements

Most jurisdictions have been willing to award for D’s negligence conduct that emotionally distresses an individual and causes a physical impact. However, most courts are less willing to permit recovery for emotional suffering alone.

Daley v. LaCroix: A plaintiff who has suffered physical injury as a result of emotional distress caused by a defendant’s negligence may recover damages, even in the absence of any physical impact upon the plaintiff. Where a definite and objective physical injury is produced as a result of emotional distress approximately caused by D’s negligent conduct, the P may recover damages not withstanding the absence of any physical impact.

Thing v. La Chusa: A parent whose child was struck by a negligent driver may not recover damages for emotional distress where the parent did not witness the accident.

5. Unborn Children

Although historically court have denied liability for fetuses, beginning in 1946, an overwhelming majority of jurisdictions have allowed a cause of action for prenatal injuries when they were inflicted on a viable fetus who was consequently born alive.

Endresz v. Friedberg: The parents of a stillborn child may not maintain a wrongful death action for the death of an unborn child.

Procanik by Procanik v. Cillo: An infant may not file a wrongful life claim to recover general damages for diminished childhood and pain and suffereing.

CH. 9 Owners and Occupiers of Land

1. Outside the Premises

Taylor v. Olsen: A landowner is under the duty to use common and reasonable methods to examine conditions on his/her property which may give rise to physical injury.

Salevan v. Wilmington Park, Inc.: A landowner who knows or should know of dangerous conditions on his/her property must take reasonable precautions to prevent injury to others using roadways adjacent to his/her land.

2. On the Premises

(A) Trespassers

Surprising to many homeowners is the fact that a duty of care is also owed to people who have no permission or lawful right to be on the premises. A trespasser is a person who enters the premises of another without express or implied permission of the owner, for the trespasser's own benefit or amusement. A homeowner cannot lawfully prepare pitfalls or traps for a trespasser in order to purposefully injure the trespasser. Once the owner is aware of the trespasser's presence or can reasonably anticipate such presence from the circumstances -- evidence of skateboarders in an unfinished swimming pool would fall into this category -- then the owner has a duty to exercise ordinary care to avoid injuring the trespasser.

Sheehan v. St. Paul & Duluth Ry. Co.: A landowner does not owe a duty of care to a trespasser of whom the landowner has no actual or constructive notice.

(B) Licensees

A licensee is a person who has no contractual relation with the property owner, but who is permitted -- expressly or implicitly -- to be on the premises. A licensee is there for his/her own interest and not for occupiers benefit. A social guest at a residence is normally considered a licensee. The homeowner is usually liable only for willful or wanton injury to a licensee. This means that the homeowner is required to exercise

Page 24: Torts Rule Statements

enough care to prevent injury to a licensee who is actually known to be -- or could reasonably be expected to be -- within the range of a dangerous act or condition.

Barmore v. Elmore: The owner/occupier of a property has the duty to warn a licensee /social guest of hidden dangers unknown to the latter.

(C) Invitees

When a homeowner invites or induces others to enter the premises for any lawful purpose, this triggers the homeowner's duty to exercise ordinary care to keep the premises safe. Invitees have an express or implied invitation to enter or use another’s premises, such as a business visitor or member of the public to whom the premises are held open. Owners want them there for their own purposes (usually financial). An invitation may be expressed (i.e. through words such as "please come in"), implied from known customs or use of the premises, or inferred from the homeowner's conduct.

Campbell v. Weathers: An individual, entering a place of business open to the public, need not make a purchase in order to be considered an invitee.

Whelan v. Van Natta: An invitee ceases to be an invitee after the expiration of a reasonable time within which to accomplish the purpose for which he is invited to enter or remain.

(D) Persons Outside the Established Categories

(1) Children

The attractive nuisance doctrine is landowner may be held liable for injuries to children trespassing on the land if the injury is caused by a hazardous object or condition on the land that is likely to attract children who are unable to appreciate the risk posed by the object or condition. The doctrine has been applied to hold landowners liable for injuries caused by abandoned cars, piles of lumber or sand, trampolines, and swimming pools. However, it can be applied to virtually anything on the property of the landowner.

According to the Restatement of Torts standard, which is followed in many jurisdictions, there are five conditions that must be met for a land owner to be liable for tort damages to a child trespasser. The five conditions of Restatement of Torts § 339 are:

A possessor of land is subject to liability for physical harm to children trespassing thereon caused by an artificial condition upon land if:

(a) the place where the condition exists is one on which the possessor knows or has reason to know that children are likely to trespass, and

(b) the condition is one of which the possessor knows or has reason to know and which he realizes or should realize will involve an unreasonable risk of death or serious bodily harm to such children,

(c) the children, because of their youth, do not discover the condition or realize the risk involved in inter-meddling with it or in coming within the area made dangerous by it

(d) the utility to the possessor of maintaining the condition and the burden of eliminating the danger are slight as compared with the risk to children involved, and

(e) the possessor fails to exercise reasonable care to eliminate the danger or otherwise to protect the children

(2) Persons Privileged to Enter Irrespective of Landowner’s Consent

(E) Rejection or Merging of Categories

Rowland v. Christian: An occupier owes a duty to warn others of known dangerous conditions which are known to the owner/occupier or to repair the conditions in order to prevent the risk of unreasonable harm to others.

Page 25: Torts Rule Statements

3. Lessor and Lessee

Borders v. Roseberry: A landlord does not, as a matter of law, owe a duty of care to a social guest of his tenant to remedy known dangerous conditions. Exceptions: (1) undisclosed dangerous conditions known to lessor and unknown to lessee, (2) conditions dangerous to persons outside the premises, (3) premises leased for admission to the public, (4) parts of land retained in lessor’s control which lessee is entitled to use, (5) where lessor contracts to repair, and (6) negligence by lessor in making repairs.

Pagelsdorf v. Safeco Ins. Co. of America: A landlord owes a tenant, or other individuals on the property with the tenant’s permission, the duty to exercise ordinary care in maintaining the premises in safe condition.

Kline v. 1500 Massachusetts Ave. Apartment Corp.: A landlord has the duty to take reasonable precautions to protect tenants from foreseeable criminal acts of their parties.

CH. 10 Damages

In any action based on negligence, the existence of actual injury is a requirement. Unlike intentional tort actions, therefore, nominal damages may not be awarded. The P must show that he suffered some kind of physical harm.

Types of Damages:

Compensatory damages: like the name suggests, are intended to compensate the injured party for his loss or injury. It is suppose to make P whole again, and restore the P to the position the P was in before the tort occurred.Punitive damages: are awarded to punish a wrongdoer. Additional sum to punish D.Nominal damages: where the court awards a nominal amount such as one dollar. A small sum awarded when a legal injury suffered but there is no substantial loss or injury to be compensated.

1. Personal Injuries

Anderson v. Sears, Roebuck & Co.: A trial court may review a jury’s award of damages to determine whether it exceeds the maximum amount that the jury could reasonably award.

Richardson v. Chapman: An award of damages will be deemed excessive if it falls outside the range of fair and reasonable compensation or results from passion or prejudice, or if it is so large that it shocks the judicial conscience.

*Economic Losses

The P can recover his direct out-of-pocket losses stemming from the injury. These include medical expenses, lost earnings, and the cost of any labor required to do things that the P can no longer do himself.*Non-economic losses

Physical pain and suffering, mental anguish: the P may also required for actual physical pain suffered from the injuries. This could included suffering before the trial as well as an estimate of suffering which will occur during the future. Also, for mental distress, the P may recover for various mental consequences of the injury (i) fright and shock at the time of the injury (ii) humiliation due to disfigurement, disability, hedonistic damages (damages for the loss of the ability to enjoy life) (iii) unhappiness and depression at being unable to lead one’s previous life and (iv) anxiety about the future.

*Judicial Control of Amounts Recovered

Page 26: Torts Rule Statements

Remittitur: a ruling by a judge (usually upon motion to reduce or throw out a jury verdict) lowering the amount of damages granted by a jury in a civil case. Usually, this is because the amount awarded exceeded the amount demanded; however, the term is sometimes used for a reduction in awarded damages even when the amount awarded did not exceed the amount demanded, but is otherwise considered excessive. If the motion is granted, the plaintiff may either accept the reduced verdict or submit to a new trial.

Additur: Additur refers to the power of a trial court to assess damages or increase a jury award of damages found to be excessively low. It is a means of avoiding a retrial where the damages awarded by the jury are deemed to be so low as to amount to a miscarriage of justice. The defendant must agree to the additur, but the plaintiff is not required to consent. It is typically involved where the jury is influenced by prejudice, corruption, passion, or mistake. It is not allowed in all states or by the federal government.

*Legislative Control of Amounts Recovered

Legislatures can and have passed laws limiting the amount of damages recoverable.

Montgomery Ward & Co., Inc. v. Anderson: Gratuitous or discounted medical services are a collateral source not to be considered in assessing the damages due a personal injury plaintiff. Collateral source rule: evidentiary rule that prohibits the admission of evidence that a victim's damages were or will be compensated from some source other than the damages awarded against the D.

Zimmerman v. Ausland: A person may not recover damages for permanent injury if the permanency could have been avoided by treatment and a reasonable person under the same circumstances would undergo the treatment.

2. Physical Harm to Property

Damages for physical harm to land or chattels is closely tied in with the concept of value, or in other words, what the property is worth. Market value is what the property in question could probably have been sold for on the open market- the highest price that could have been realized. The rule of highest intermediate value allows as damages the highest value which goods have reached during the period from time of the wrong to trial. The New York rule is the highest market price between time of conversion and reasonable period.

3. Punitive Damages

Cheatham v. Pohle: To the extent punitive damages are recoverable, they are a creature of the common law. The legislature is free to create, modify, or abolish common law causes of action. And, as a matter federal constitutional law, no person has a vested interest or property right in a rule of common law. The award to the Fund is not the property of the plaintiff. Nor is her prejudgement claim a property interest. Therefore, because the punitive damages do not compensate the P, the P has no right or entitlement to an award in punitive damages in any amount. Any punitive damage given to P is a creation of state law, which means the money taken by the P is not the property of the P, nor a property interest.

State Farm Mutual Automobile Ins. Co. v. Campbell: The Court held that the punitive award of $145 million was neither reasonable nor proportionate to the wrong committed, and it was thus an irrational, arbitrary, and unconstitutional deprivation of the property of the insurer. The Court reasoned that evidence of dissimilar out-of-state misconduct was an improper basis for punishing the insurer for the limited harm and noted, "few awards exceeding a single-digit ratio between punitive and compensatory damages, to a significant degree, will satisfy due process."

CH. 11 Wrongful Death and Survival

Page 27: Torts Rule Statements

The difference between a wrongful death statute and a survival statute is that a survival action permits recovery for the wrong to the injured person and is confined to his personal loss while the wrongful death action is for the wrong to the beneficiaries, confined to their loss because of the death. The latter begins where the former ends and recovery on both is not double recovery for a single wrong but rather separate recoveries for different wrongs.

1. Wrongful Death

Wrongful death statutes create a cause of action for any wrongful act, neglect, or defect that causes death; the action may be brought by the executor or administrator of the decedent’s estate or by his surviving family and is expected to compensate the family for the loss of economic benefits that would have had in the form of support, services, or contributions had the decedent lived. Recovery for wrongful death was held to cover damages for (1) loss of support including all financial support that the D would have made to his dependants had he lived, (2) the monetary value of services that the decedent provided and would have continued to provide but for his wrongful death (i.e. nurture, training, education and guidance and services the decedent performed at home for his spouse), (3) compensation for loss of society (including love, attention, care, affection, companionship, comfort…) (4) damages for funeral expenses.

Morangne v. States Marine Lines, Inc.: Maritime law should allow a cause of action for wrongful death.

Selders v. Armentout: The measure of damages for the wrongful death of a minor child should include loss of society, comfort and companionship of the child.

2. Survival

Survival statute which preserves for a decedent’s estate a cause of action for infliction of pain and suffering and related damages suffered up to the moment of death by the decedent. Physical and mental pain and suffering are of the decedent are allocated to the survival action, as are medical expenses. As well as decedent’s loss of potential earnings during his life expectancy.

Murphy v. Martin Oil Co.: The spouse of a decedent may maintain an action for the loss of property, loss of wages, and pain and suffering of the decedent during the interval between the injury and death.

CH. 12 Defenses

1. Plaintiff’s Conduct

(A) Contributory Negligence

Contributory negligence is conduct on the part of the plaintiff that falls below the standard to which he should conform for his own protection, and which is a legally contributing cause in addition to the negligence of the defendant in bringing about the plaintiff’s harm. At common law, any amount of contributory negligence barred recovery by the plaintiff. Basically, it is a doctrine of common law that if a person was injured in part due to his/her own negligence (his/her negligence "contributed" to the accident), the injured party would not be entitled to collect any damages (money) from another party who supposedly caused the accident. Under this rule, a badly injured person who was only slightly negligent could not win in court against a very negligent defendant.

Butterfield v. Forrester: An injured plaintiff may not recover damages against a negligent defendant if the plaintiff did not exercise reasonable care to avoid the injury.

Davis v. Mann: A negligent defendant, claiming contributory negligence on the part of the plaintiff, may not escape liability if the defendant had the last chance to prevent the injury had he/she not been negligent. (Last clear chance doctrine: A doctrine that a defendant may be liable for the injury he

Page 28: Torts Rule Statements

caused even though the plaintiff was contributorily negligent, if the defendant could have avoided the injury to the plaintiff by exercising ordinary care.)

(B) Comparative Negligence

Comparative negligence is the allocation of responsibility for damages incurred between the plaintiff and defendant, based on the negligence of the two; the reduction of the damages to be recovered by the negligent plaintiff in proportion to his fault.

Prior to the late 1960s, however, only a few states had adopted this system. When comparative negligence was adopted, three main versions were used. The first was called "pure" comparative negligence. A plaintiff who was, say, 90% to blame for an accident could recover 10% of his losses. (Of course, if the defendant suffered injuries in such a case, he/she could counter claim and recover 90% of his/her losses from the other party.)

The second and third versions are lumped together in what is called "modified" comparative negligence. One variant allows plaintiffs to recover only if the plaintiff's negligence is "not greater than" the defendant's (viz., the plaintiff's negligence must not be more than 50% of the combined negligence of both parties).

The other variant allows plaintiffs to recover only if the plaintiff's negligence is "not as great as" the defendant's (viz., the plaintiff's negligence must be less than 50% of the combined negligence). The apparently minor difference between the two modified forms of comparative negligence are thought by lawyers handling such cases to be significant in that juries who ordinarily assign degrees of fault are much less willing to award damages to a plaintiff who is equally at fault than to one who is less at fault than the defendant.

McIntyre v. Balentine: The doctrine of contributory negligence should be replaced by a system of comparative fault to allow the plaintiff, whose negligence is less than the defendant, to recover damages which are reduced in proportion to the plaintiff’s percentage of fault.

(C) Assumption of Risk

Assumption of risk which bars a plaintiff from recovery against a negligent tortfeasor if the defendant can demonstrate that the plaintiff voluntarily and knowingly assumed the risks at issue inherent to the dangerous activity in which he was participating at the time of his injury.

(1) Express

If the P expressly agrees with the D, in advance of any harm, that the P will not hold D liable for certain harm, the P is said to have “expressly” assumed the risk of that harm. If there is no “public policy” against the assumption of risk involved, the agreement will be enforced and the P may not recover.

Seigneur v. National Fitness Institute, Inc.: Public interest will render an exculpatory clause unenforceable when: (1) the party protected by the clause intentionally causes harm or engages in acts of reckless, wanton, or gross negligence; (2) the bargaining power of one party to the contract is so grossly unequal so as to put that party at the mercy of the other's negligence; and (3) the transaction involves the public interest. The court places itself in the position of the parties at the time they made the contract, so as to view the circumstances at the time and to judge the meaning of the words in the contract in context. None of the three conditions apply in this case, so the clause is upheld and the club is released from liability. The ultimate determination of what constitutes the public interest must be made considering the totality of the circumstances of any given case against the backdrop of current societal expectations.

Page 29: Torts Rule Statements

(2) Implied

Even if the P never makes an actual agreement with the D whereby the risk is assumed by the former, she may be held to have assumed certain risks by her conduct. In this situation the assumption of risk is said to be implied. The P must have knowledge of the risk (1) it must be shown that the particular risk in question was known to the D (2) the risk was one which was actually known to the P (3) however, in some situations P indicates consent for unknown risks.

Primary assumption of risk- the D not negligent either because he owed no duty to the P in the first place or because he did not breach the duty owed. Secondary assumption of risk- affirmative defense to an established breach of a duty owed by the D to the P.

Rush v. Commercial Realty Co.: A person does not voluntarily accept a risk when no reasonable alternatives exist.

Blackburn v. Dorta: Implied assumption of risk is subsumed by the doctrine of comparative negligence.

2. Statutes of Limitations and Repose

When the P does not discover his injury until long after the D’s negligent act occurred, does the statute of limitations start to run at the time of the act, or at the time of the discovery?

Teeters v. Currey: A cause of action for malpractice does not accrue, and the statute of limitations does not begin to run, until the injured party discovers or should have discovered the injury. Under the discovery doctrine, the statute does not begin to run until the negligent injury is or should have been discovered. The traditional rule is a harsh and oppressive rule. 28 states now embrace the discovery doctrine.  In those classes of cases where medical malpractice is asserted to have occurred through the negligent performance of surgical procedures, the cause of action accrues and the statute of limitations commences to run when the patient discovers or in the exercise of reasonable care and diligence for his own health and welfare, should have discovered the resulting injury. 

3. Immunities

An immunity is a defense to tort liability that is given to an entire class of persons based on their relationship with the prospective P, the nature of their occupation, the status as a governmental or charitable entity. The common law created a number of virtually complete immunities, but all of these are beginning to break down at least to some extent, either by statutory reform or judicial overruling.

(A) Families

Husband and wife: At common law, the husband and wife were considered as one person. Now, over half the states have completely abolished the inter-spousal immunity, even for personal injury suits. In states that have not completely abolished the immunity, a number of limitations on it are commonly applied.

Freehe v. Freehe: Spouses are not immune from liability in personal injury cases. Rejected all traditional reasons for the immunity including: (1) oneness of husband and wife, (2) peace and tranquility (if there is such peace and tranquility the situation is such that either no action will be commenced or that the parties will allow the action to continue so long as their personal harmony is not jeopardized) (3) flood of litigation (has not materialized in other states that have previously abolished the immunity) and (4) collusion and fraud.

Page 30: Torts Rule Statements

Parent and child: Common law barred suit from a child against his parents or vice versa (exception violence against the child). About a third of the states have now abolished this immunity, at least in automobile accident suits. One factor that is frequently stressed is that nearly everyone has liability insurance.

Renko v. McLean: There is no exception to the parent-child immunity doctrine for injuries sustained in motor tort cases occurring during the child’s minority.

(B) Charities

Charitable organizations, as well as educational and religious ones, received immunity at common law. Reasons given for this (1) trust fund- the charity holds the donations it receives in trust, and the donor has not given these funds with the intention that they be used to pay tort claims, (2) implied waiver- the beneficiary of charity has impliedly waived his right to sue in tort, by virtue of having accepted this benevolence. However, by now, more than thirty states have abolished charitable immunity.

Abernathy v. Sisters of St. Mary’s: Nongovernmental charitable institutions are liable for their own negligence and the negligence of their employees. Relied on the availability of liability insurance in abolishing the defense, although the existence of such insurance in any particular case is irrelevant to liability. In reference to the reasons given to favor this immunity, (1) the court said the trust fund-liability should not be determined based on whether the charity can satisfy a judgement, and (2) in terms of the implied waiver- there are some people who could not have possibly waived their right to sue, such as an unconscious accident victim carried to a charitable hospital, meaning implied waiver is a legal fiction.

(C) Employer Immunity

Since statutes allow employees to recover from their employers for work-related injuries without having to show any fault on the part of the employer- an employee who is injured on the job generally cannot file a tort claim against his employer even if employer was negligent.

(D) State and Local Governments

At English common law, sovereign immunity/ immunity of the king developed. The doctrine, which was connected to the divine rights of kings, was sometimes expressed by saying that the king can do no wrong.

Early American courts applied the English rule to hold that the United States could not be sued without its consent. The first major and meaningful consent by the United States to torts claims was embodied in the 1946 Federal Torts Claims Act (FTCA). The act continues today to be dispositive of almost all possible tort claims against the government. Today, almost all states have limited sovereign immunity to some extent. Many states and the federal government continue immunity for discretionary functions (those where the government is acting to establish policy, i.e. how much money to spend on police) but have eliminated it for ministerial acts (those that implement or effectuate the policies i.e. police officer negligently driving car).

Ayala v. Philadelphia Board of Public Education: Governmental entities are not immune from tort liability.

Riss v. New York: Cities are immune from liability for the negligent failure to provide adequate police protection.

DeLong v. Erie County: Municipalities are not immune from liability when they voluntarily assume a duty to provide police protection for a particular individual.

(E) The United States

Deuser v. Vecera: Discretionary conduct of government agents acting within the scope of their authority are protected by the discretionary function exception to the Federal Torts Claims Act.

Page 31: Torts Rule Statements

(F) Public Officers

Public officers can be held individually liable for torts committed in the course of their official duties. Public officials have limited common-law immunities: (a) legislators and judges have been granted immunity for acts within the scope of their office (b) the president of the U.S. is absolutely immune for acts within the scope of his office, but presidential aides are only entitled to qualified immunity, (c) some states officers have official immunity, but only for acts that are discretionary in nature.

CH. 13 Vicarious Liability

Vicarious liability is when one person is liable for the negligent actions of another person, even though the first person was not directly responsible for the injury.

1. Respondent Superior

Respondent superior is a common-law doctrine that makes an employer liable for the actions of an employee when the actions take place within the scope of employment.

Bussard v. Minimed, Inc.: Foreseeability as a test for respondeat superior merely means that in the context of the particular enterprise an employee's conduct is not so unusual or startling that it would seem unfair to include the loss resulting from it among other costs of the employer's business. Such a test is useful because it reflects the central justification for respondeat superior: that losses fairly attributable to an enterprise- those which foreseeably result from the conduct of the enterprise- should be allocated to the enterprise as a cost of doing business. This means the employer could take precautions (like in this case where they asked the employee if she was capable of driving home) and still be liable.

O’Shea v. Welch: If an employee wholly abandons, even temporarily, the employer's business for personal reasons, the act is not within the scope of employment, and the employer is not liable under respondeat superior for the employee's conduct during that lapse. Slight deviation rule: it must be determined whether the employee was on a frolic (abandonment of employer’s business while in pursuit of employee’s own personal business) or a detour (slight deviation from employer’s own business for employee’s own reasons).

2. Independent Contractors

The employer of an independent contractor is generally not held vicariously liable for the tortious acts and omissions of the contractor, because the control and supervision found in an employer-employee or Principal-Agent relationship is lacking. However, vicarious liability will be imposed in three circumstances:

1. the contractor is involved in an ultra-hazardous activity (one likely to cause substantial injury, such as blasting with explosives)

2. The employer is herself negligent in her dealing with the independent contractor.3. Non-delegable duty, or duties of care that are deemed so important that the person doing them will

not be allowed to delegate them to anyone. Murrell v. Goertz: Employer is not liable for the torts of his independent contractor. Maloney v. Rath: A motorist cannot delegate her duty to keep her brakes in working order to an

independent contractor.

3. Joint Enterprise

Page 32: Torts Rule Statements

A joint enterprise is like a partnership, except that it is generally for a very short and specific purpose. Once the various requirements for joint enterprise are met, the negligence of one joint enterpriser is imputed to the other.

Requirements of joint enterprise: (1) an agreement express or implied between the parties, (2) a common purpose to be carried out by the members, (3) a common pecuniary interest in that purpose and (4) and equal right to a voice in the enterprise, i.e. right of control.

Popejoy v. Steinle: A defendant is not vicariously liable for the negligence of another under a theory of joint venture unless the joint venture was motivated by profit.

4. Bailments

A legal relationship created when a person gives property to someone else for safekeeping. To create a bailment, the other party must knowingly have exclusive control over the property. The receiver must use reasonable care to protect the property.

Shuck v. Means: Expresses modern view. A car rental agency is liable for the negligence of an individual who drives its rental car, even though it did not rent the car to that individual. Proving lack of consent in these situations requires a strong showing that the car was being used by the permittee without the owner's knowledge and contrary to his explicit instructions, or that the subpermittee was driving without the permission of the first permittee under conditions which approach the status of conversion or a theft.

5. Imputed Contributory Negligence

Imputed contributory negligence has acquired a bad reputation in the law. Traditionally, imputed contributory negligence has been when a person (A) is vicariously liable for the tortuous actions of another (B) and a third person (C) brings an action, and that makes B’s contributory negligence impute to A when A sues C. That doctrine caused in many cases an entirely innocent person, who was seriously injured by no fault of their own, be barred from recovery against another a person who negligently injured them, because they were charged with the negligence of another who had contributed to the injury.

Historically, there have been several areas in which contributory negligence was imputed: (1) Driver and Passenger- A driver’s negligence was frequently imputed to his passenger, so as to prevent the passenger from recovering against the driver of another vehicle whose negligence contributed to the collision between the two cars. This resulted in making the entire loss of the accident fall upon the one person who was free from negligence. (2) Husband and wife: A husband’s negligence was frequently imputed to his wife and vice versa. Today, the contributory negligence of one spouse is no longer imputed to bar a recovery by the other. (3) Parent and child- A parent’s negligence was imputed to his child.

Modern rule: In most states, contributory negligence will be imputed only if the relationship between is one which would make the P vicariously liable if he were a D (contributory negligence could not be imputed unless negligence could be imputed).

Both ways test: The formerly accepted test that if the relationship is one which would give rise to vicarious liability, contributory negligence must be imputed (if negligence could be imputed, contributory negligence will also be).

Smalich v. Westfall: Contributory negligence may not be imputed to a passenger riding their own automobile without the finding of either a master-servant relationship or a finding of joint enterprise. A plaintiff ought not to be barred from recovery against a negligent defendant by the contributory

Page 33: Torts Rule Statements

negligence of a third person unless the relationship between the plaintiff and the third person is such that the plaintiff would be vicariously liable as a defendant for the negligent acts of the third person.

CH. 14 Strict Liability

1. Animals

Trespassing animals: In most states, the owner of livestock or other animals is liable for property damage caused by them if they trespass on another’s land. This liability is strict even if the owner exercises utmost care to prevent the animals from escaping, he is liable if they do escape and trespass.

A person is also strictly liable for all damage done by any dangerous animal he keeps.

Wild animals: A person who keeps a wild animal is strictly liable for all damage done by it, as long as the damage results from the dangerous propensity that is typical of the species in question.

However, injuries caused by domestic animals such as dogs or cats, do not give rise to strict liability unless the owner knows or has reason to know that of the animal’s dangerous propensities/characteristics.

2. Abnormally Dangerous Activities

Abnormally dangerous activity is an undertaking that necessarily carries with it a significant risk of serious harm even if reasonable care is used, and for which the actor may face strict liability for any harm caused. An actor may be strictly liable because (1) involves the risk of serious harm to persons or propery, (2) cannot be performed without this risk regardless of the precautions taken and (3) does not ordinarily occur in the community. The path to strict liability for abnormally dangerous activities was begun in the English case of Rylands v. Fletcher. In law, there is a difference between things that grow or occur naturally on the land, and those that are accumulated there artificially by the defendant. For example, rocks and thistles naturally occur on land. However, the defendants in Rylands v Fletcher brought water onto the land.

Restatement § 519 states the general principle for liability, and § 520 provides several evaluative factors. Section 519 provides for strict liability for one “who carries on an abnormally dangerous activity” causing harm to persons or property even if he “has exercised the utmost care to prevent the harm.” Section 520 suggests evaluative factors to assist in determining if an activity should be termed abnormally dangerous (ultrahazardous), and includes (1) the existence of a high degree of risk of some harm; (2) the magnitude of that harm (likelihood that the harm will be great); (3) the inevitability of some risk irrespective of precautionary measures that might be taken; (4) the ordinary or unusual nature of the activity (extent to which the activity is of common usage); (5) inappropriateness of the activity to the place where it is carried on and (6) the activity's value to the community in comparison to the risk of harm created by its presence.

Rylands v. Fletcher: A person carrying on an abnormally dangerous activity is absolutely liable for any damage caused by that activity, even though he was not negligent, that principle applies only to a "nonnatural" use of the defendant's land, as distinguished from "any purpose for which it might in the ordinary course of the enjoyment of land be used." Strict liability exists for harm resulting from the miscarriage of lawful activity that, considering its place and manner, is unusual, extraordinary, or inappropriate.

Miller v. Civil Constructors, Inc.: The essential question is whether the risk created is so unusual, either because of its magnitude or because of the circumstances surrounding it, as to justify the imposition of strict liability even though the activity is carried on with all reasonable care. The use of guns or firearms, even though classified as dangerous or even highly dangerous, is not the type of activity that must be deemed ultrahazardous.

Page 34: Torts Rule Statements

Indiana Harbor Belt R.R. Co. v. American Cyanamid Co.: The manufacture and shipping (as opposed to carrying) of toxic chemicals is not abnormally dangerous.

3. Limitations on Strict Liability

Foster v. Preston Mill Co.: A person carrying on an abnormally dangerous activity is not strictly liable for damage that is not within the scope of danger created by that activity.

Golden v. Amory: A person carrying on an abnormally dangerous activity is not absolutely liable for if damage caused by that activity was precipitated by an act of God.

Sandy v. Bushey: The owner of an animal known to the owner to be vicarious is strictly liable for damage caused by the animal. Contributory negligence is not a defense to strict liability.

CH. 15 Products Liability

Products liability is the umbrella term for the liability of a manufacturer, seller, or other supplier of chattels, to one with whom he is not in privity of K., who suffers physical harm caused by the chattel.

1. Development of Theories of Recovery

(A) Negligence

The general rules of negligence apply to one who sells a product. Most commonly negligence theory is used to make a manufacturer liable for where he failed to use reasonable care in designing, manufacturing, or labeling the product.

MacPherson v. Buick Co.: When there is an element of probable danger with a product, the manufacturer is under a duty to make it carefully. This case established the general principle that once the P shows that the product will be unreasonably dangerous if defective, he may sue in negligence without privity. Every state has accepted this case, meaning one who negligently manufactures a product is liable for any personal injuries proximately caused by his negligence.

(B) Warranty

Freak hybrid: “born of the illicit intercourse of tort and contract”

(1) Express Warranties

An express warranty is a guarantee from the seller of a product that specifies the extent to which the quality or performance of the product is assured and states the conditions under which the product can be returned, replaced, or repaired. It is often given in the form of a specific, written "Warranty" document. However, a warranty may also arise by operation of law based upon the seller's description of the goods, and perhaps their source and quality, and any material deviation from that specification would violate the guarantee. An express warranty can be made orally, in writing and without the intent of the seller to actually create the warranty. In the United States, a seller is allowed to assert statements of opinion of value, known as puffery, that the buyer cannot justly rely on as part of the basis for the bargain.

Baxter v. Ford Motor Co.: The breach of an express warranty is actionable in tort, even absent privity of contract, if a purchaser of ordinary experience and reasonable prudence could not have discovered defect.

(2) Implied Warranties

An implied warranty is one that arises from the nature of the transaction, by putting good in stream of trade, and the inherent understanding by the buyer, rather than from the express representations of the seller.

Page 35: Torts Rule Statements

The warranty of merchantability is a warranty implied by law that goods are reasonably fit for the general purpose for which they are sold, unless expressly disclaimed by name, or the sale is identified with the phrase "as is" or "with all faults." To be "merchantable", the goods must reasonably conform to an ordinary buyer's expectations, i.e., they are what they say they are.

The warranty of fitness for a particular purpose is implied when a buyer relies upon the seller to select the goods to fit a specific request.

Henningsen v. Bloomfield Motors, Inc.: A contract of adhesion does not trump statutory implied warranties of merchantability. This case drastically restricted the privity requirement in warranty cases in almost exactly the same way as McPherson had restricted the privity requirement for negligence actions. All states have now accepted the holding in this case, meaning that they hold a manufacturer’s warranty extends to remote purchasers further down the line, and they also hold that once the final purchaser is covered by the warranty, the warranty also applies to at least members of the household who may reasonably be expected to use the goods.

(C) Strict Liability in Tort

Greenman v. Yuba Power Products, Inc.: Manufacturers are strictly liable for injuries caused by defective products if the user does not know of the defects. A manufacturer is strictly liable in tort when an article he places on the market, knowing that it is to be used without inspection for defects, proves to have a defect which causes injury to a human being.

R2 § 402A Special Liability of Seller of Product for Physical Harm to User or Consumer:(1) One who sells any product in a defective condition unreasonably dangerous to the user or consumer or to his property is subject to liability for physical harm thereby caused to the ultimate user or consumer, or his property if(a) the seller is engaged in the business of selling such a product, and(b) it is expected to and does reach the user or consumer without substantial change in the condition in which it is sold.

(2) The rule stated in subsection (1) applies although (a) the seller has exercised all possible care in the preparation and sale of his product, and(b) the user or consumer has not bought the product from or entered into any contractual relation with the seller.

* § 402A applies also to the product’s retailer, and any other person in the distributive chain (e.g. a wholesaler).

Restatement Third of Torts: Product Defective Products

§ 1. Liability of Commercial Seller or Distributor for Harm Caused by Defective Products

(a) One engaged in the business of selling or otherwise distributing products who sells or distributes a defective product is subject to liability for harm to persons or property caused by the defect.

§ 2. Categories of Product Defect

A product is defective when, at the time of sale or distribution, it contains a manufacturing defect, is defective in design, or is defective because of inadequate instructions or warnings. A product:

Page 36: Torts Rule Statements

(a) contains a manufacturing defect when the product departs from its intended design even though all possible care was exercised in the preparation and marketing of the product;(b) is defective in design when the foreseeable risks of harm posed by the product could have been reduced or avoided by the adoption of a reasonable alternative design by the seller or other distributor, or a predecessor in the commercial chain of distribution, and the omission of the alternative design renders the product not reasonably safe;(c) is defective because of inadequate instructions or warnings when the foreseeable risks of harm posed by the product could have been reduced or avoided by the provision of a reasonable instructions or warnings by the seller or other distributor, or a predecessor in the commercial chain of distribution, and the omission of the instructions or warnings renders the product not reasonably safe.

2. Product Defects

Defective products can cause serious injury and even death. Defects can be traced to three main stages: when the product is designed, when the product is manufactured and when the consumer should receive instructions or warnings.

(A) Manufacturing Defect

When a manufacturing defect occurs, it can happen despite careful design. No matter how exacting the planning, the process can still break down during manufacturing. Even if the quality control is reasonable, the manufacturer is still at fault if, for instance, the product has a weak spot, a crack or another flaw.A manufacturer that produces a product with a manufacturing defect faces the strict liability standard. This means that no matter what safety steps the manufacturer took during the production process, it is at fault if the product causes injury due to a manufacturing defect. This standard encourages manufacturers to be vigilant during the manufacturing process, and it eases the plaintiff's burden of proof.

Rix v. General Motors Corp.: Sellers are strictly liable for manufacturing defects that reach the consumer without substantial change in the defective condition.

(B) Design Defect

Design defects are in a manner of speaking, intended. This type of defect is inherent in the design of the product. A design defect occurs in the infancy of a product. It is a fundamental flaw that makes the product unsafe. If a consumer uses the product in the intended manner (or in a foreseeable manner), and the consumer is injured by the product, then the consumer may be able to recover compensation.The injured plaintiff must show that the harmful product was defectively designed. Depending on the state in which the legal action takes place, this will mean proving that the design was unreasonably dangerous or the design was negligent. The plaintiff also may need to show that a safer alternative design was available and feasible.

Prentis v. Yale Mfg. Co.: Design defects cases should be judged under a negligence test, weighing the risks of injury against the costs of safer designs. Risk utility test: a test used in product liability cases to determine whether a manufacturer is liable for injury to a consumer because the risk of danger created by the product's design outweighs the benefits of the design.

O’Brien v. Muskin Corp.: A plaintiff must make a prima facie showing of a defective product, based on the risk utility analysis. Generally speaking, a plaintiff has the burden of proving that (1) the product was defective; (2) the defect existed when the product left the hands of the defendant; and (3) the defect caused injury to a reasonably foreseeable user. In a design-defect case, the plaintiff bears the burden of both going forward with the evidence and of persuasion that the product contained a defect. To establish a prima facie case, the plaintiff should adduce sufficient evidence on the risk-utility factors to establish a defect.

Page 37: Torts Rule Statements

Some factors relevant in risk-utility analysis are:

(1) The usefulness and desirability of the product -- its utility to the user and to the public as a whole.

(2) The safety aspects of the product -- the likelihood that it will cause injury, and the probable seriousness of the injury.

(3) The availability of a substitute product which would meet the same need and not be as unsafe.

(4) The manufacturer's ability to eliminate the unsafe character of the product without impairing its usefulness or making it too expensive to maintain its utility.

(5) The user's ability to avoid danger by the exercise of care in the use of the product.

(6) The user's anticipated awareness of the dangers inherent in the product and their avoidability, because of general public knowledge of the obvious condition of the product, or of the existence of suitable warnings or instructions.

(7) The feasibility, on the part of the manufacturer, of spreading the loss by setting the price of the product or carrying liability insurance.

(C) Warnings Defect

Even when a product has been properly designed and manufactured, it still may not be safe for all uses. Manufacturers and sellers must take adequate steps to avoid unreasonable risk to consumers. This means that when a product could be dangerous, manufacturers must warn the consumer of dangers that are not obvious and instruct on proper use. If a satisfactory warning is in a prominent or proper location and the consumer fails to read it, then the consumer typically may not later collect damages from the manufacturer for failing to provide adequate warning. If, however, the warning is absent, hard to see or unclear, then the consumer may have a viable case. The manufacturer's failure to warn must be the cause of the consumer's injuries.

Anderson v. Owens-Corning Fiberglass Corp.: Evidence that a particular risk was neither known nor knowable by the application of scientific knowledge available at the time of manufacture and/or distribution provides a defense to warnings defect cases. California cts, either expressly or by implication, have to date required knowledge, actual or constructive, of potential risk or danger before imposing strict liability for a failure to warn.  State of the art evidence may be relevant to the question of knowability and should be admissible.  Exclusion of the evidence would make the manufacturer a virtual insurer of the products safety.  The warning defect relates to a failure extraneous to the product itself.  While a manufacturing or design defect can be evaluated w/o reference to the conduct of the manufacturer, the giving of a warning cannot b/c that warning requires a communication of something to someone.  Strict liability is not concerned w/ the standard of due care or the reasonableness of a manufacturer’s conduct. The fact that a manufacturer acted as a reasonably prudent manufacturer in deciding not to warn, although perhaps absolving the manufacturer of liability under a negligence theory, would not preclude liability under strict liability principles if the trier of fact concluded that, based on information scientifically available to the manufacturer, the manufacturer's failure to warn rendered the product unsafe to its users.

Note: Warnings is one factor to consider in the risk utility analysis, but warnings alone will not save a product from being unreasonably dangerous. When reasonable minds may differ as to whether a risk is obvious (no warning required for obvious risks) or generally known, the jury decides.

3. Proof

Page 38: Torts Rule Statements

Three things P must prove: (1) the product that injured the P was in fact manufactured by D, (2) product was defective and P was injured as a result, and (3) the defect was present in the product at the time of the sale.

Friedman v. General Motors Corp.: Circumstantial evidence of defects may be used to make a prima facie case. A defect may be proven by circumstantial evidence, where a preponderance of that evidence establishes that the accident was caused by a defect and not other possibilities, although not all other possibilities need be eliminated.

4. Defenses

(A) Plaintiff’s Conduct

Daly v. General Motors Corp.: Comparative fault principles can be applied to strict products liability actions. The P’s injury must have been caused by a “defect,” in the product.  The manufacturer is not deemed responsible when injury results from an unforeseeable use of its product. Therefore, loss should be assessed equitably in proportion to fault. Comparative negligence is a defense to strict liability. Strict liability was imposed against manufacturers to relieve consumers “from the problems of proof inherent in pursuing negligence and warranty remedies.”  Cts have sought to place the burden of loss on manufacturers rather than “injured persons who are powerless to protect themselves.” P’s own conduct, relative to the product, is not allowed to escape unexamined, and as to that share of P’s damages which flows from his own fault there is no reason why it should be borne by others.  A system of comparative fault should be and is hereby extended to actions founded on strict products liability.  Principles of comparative negligence expressed in this decision apply to actions founded on strict products liability, thereby reducing plaintiff's recovery only to the extent that his own act of reasonable care contributed to his injury; in such cases, there exists no separate defense of "assumption of risk" to the extent that such assumption of risk is a form of contributory negligence.The separate defense of assumption of risk in its form of contributory negligence is abolished.

Ford Motor Co. v. Matthews: Manufacturers are liable for reasonably foreseeable, albeit abnormal, uses of products. Although misuse of a product that causes an injury is normally a bar to strict liability, it is said that “The manufacturer is not liable for injuries resulting from abnormal or unintended use of his product, if such use was not reasonably foreseeable.  The issue is one of foreseeability and misuse may be foreseeable.  These are factual issues of foreseeability of a particular use that are left to the jury.  The tractor was designed to prevent its starting in gear.  It could be foreseeable by Ford that one day a tractor operator might carelessly crank the engine w/o first making certain that it was not in gear, especially if he were aware of the purpose of the safety switch system.  Even if Pl were negligent, such negligence was reasonably foreseeable by Ford and is not a bar to an action based on strict liability resulting from a defective tractor.

(B) Preemption and Other Government Actions

Medtronic, Inc. v. Lohr: The Medical Device Amendments of 1976, a federal statute, does not preempt a state common-law negligence action against the manufacturer of an allegedly defective medical device.

5. Defendants Other than Principal Manufacturers/ Harm Other than Personal Injury

(A) Other Suppliers of Chattels

Not every party in the distribution chain is necessarily subject to strict products liability. The policy rationale is that remote distributors are in no position to exert pressure on manufacturers to create safe products.

Page 39: Torts Rule Statements

Peterson v. Lou Bachrodt Chevrolet Co.: A remote retailer, who outside of the original producing and marketing chain, is not subject to strict products liability.

(B) Services

Providers of services generally are not subject to strict products liability. Even where the services involve the provision of a product, such as a pacemaker, the service provider is not an integral part of the distribution process and except.

Hector v. Cedars-Sinai Medical Ctr.: Providers of medical services are not subject to strict products liability.

(C) Harm Other Than Personal Injury

(1) Economic loss resulting from personal injury (i.e. lost wages) are recoverable in strict liability actions, (2) economic loss w/o personal injuries is limited (3) unless the alleged defect creates an unreasonable risk of injury to persons, even though no one was actually injured (4) courts allow for damage to property under strict liability.

6. Legislation and Products Liability

Many states adopted statutes similar to the Restatement (Second) of Torts § 402A in the 1960’s.With tort reform in the 1970’s, states passed laws restricting the scope of products liability in order to protect manufacturers and reduce liability insurance premiums. The federal government proposed a Uniform Products Liability Act in 1977, which was adopted by some states and followed by courts in other states in creating common law. More tort reform legislation was passed in the 1980’s and 1990’s in further efforts to reduce insurance rates, products manufacturers and promote fairness.